You are on page 1of 109

ENGLISH LANGUAGE

 English Grammar
 Reading Comprehension
 Correct Sentences
 Vocabulary

English Grammer:-

Noun:
To put it very simply, a noun is nothing but a name. Anything that is a name, is actually a noun! Here we will
learn some rules and guidelines about how to use nouns correctly and in the proper form. And also look at
some questions that are often asked in bank exams. Let us get started.

What is a Noun?
A noun is anything that is an individual or a thing, a place, or even a feeling or idea. Basically, the noun is
any word that is used to identify a particular entity. Let us understand it better with the following
examples.

Examples of Noun
Name The teacher, the doctor, Mr. Jones, the banker, Mrs. Ray.
Thing: A ball, the swing, an umbrella.
Place: Classroom, Mumbai, the road, Canada.
Feeling: Concern, worry, beauty.
Action: Exchange, running, sleeping, breathing.

Singular and Plural Nouns


One thing you must remember is how to make singular nouns into plural ones. Singular noun refers to one
single aspect, whereas plural is a reference to a collection of things (more than one). Let us take a look at a
few pointers to keep in mind

 Generally in most cases to turn a singular noun to a plural noun we just add the letter “s” to the end.
The tree becomes trees and cat becomes cats.
 If the noun ends in s, ss, sh, o, x or z then the plural noun will end with an “es”. Crunch will become
crunches, and mass becomes masses.
 If the noun ends with f or fe the plural nouns will end with “ves”. So plural of wife will be wives and
calf will be calves.
 And if the noun will end with a y, then the ending of the plural nouns will be “ies’. Puppy in its plural
form will be puppies. But if the letter before y is a vowel then you will only add a “-s”. So boy will be
boys
Points to Keep in Mind
There are also some important rules or guidelines that you must keep in mind regarding nouns, that will
be useful.

1. When dealing with a collective noun, we use gender-neutral pronouns. For example, “The class was
doing their work in silence.” We do not use the pronouns his or her.
2. If it is an animal or creature or even a baby, again we use gender-neutral pronouns.
3. There are certain words that sound plural, but are actually singular nouns and used as such in
a sentence. Some such nouns are Mathematics, Politics, Ethics, Optics, Economics etc. So, for
example, Mathematics is a hard subject for Ricky. Notice we used “is and not “are”.
4. As opposed to that there are nouns that sound singular but are actually are plural. Some such nouns
are Children, Cattle, Women. Men etc. The Women are walking towards the market. Again we used
“are” since women is a plural noun.
5. And then there are a few fords whose plural and singular noun is the same. Words like sheep, deer,
fish, aircraft, team etc can be used as both singulars as well as plural nouns.
Learn Kinds of Nouns here.

Solved Questions

Type 1 Activity
Q: Every evening(1) the mothers-in-law(2) would meet to discuss their respective days(3). Which part of
the sentence is incorrect?

a. 1
b. 2
c. 3
d. No errors in the sentence.
Ans: The correct answer is “B”. The plural form of mother-in-law would be mothers-in-law and not
mothers-in-law.
Q: The laugh (1) was so loud (2) it could be heard from across the room (3). Which part of the sentence is
incorrect?

a. 1
b. 2
c. 3
d. No errors in the sentence.
Ans: The correct answer is “A”. The noun form of “laugh” (which is a verb) must be used here, which is
laughter.
Q: The deer (1) were terrified (2) and ran away from the car (3). Which part of the sentence is incorrect?

a. 1
b. 2
c. 3
d. No errors in the sentence
Ans: The correct answer is “D”. The plural form of deer is also deer, and the sentence is correctly formed.

Type 2 Activity

Q: ______ of people met at the ground and got to work.

a. Hundred
b. Hundreds

Ans: Correct answer is “B”. People is a plural noun by itself. So the correct sentence is “Hundreds of people
met at the ground and got to work. ”
Q: Will you at least ____ the job?

a. Consider
b. Considered
Ans: The correct answer is “A’. Consider is the noun, and considered is the verb form. Here the correct
sentence will be “Will you at least consider the job?

Practice Question
Q: Woman and men (1) should be treated (2) by society equally (3). Which part of the sentence is
incorrect?

a. 1
b. 2
c. 3
d. No errors in the sentence.
Ans; Option A. Should be ‘men’.
Q: It was one of the most enjoyable experiences of his life. Which part of the sentence is incorrect?

a. 1
b. 2
c. 3
d. No errors in the sentence.
Ans: Option D. ‘Experience’ can be both singular and plural.
Q: He was certain that the ____ was false.

a. Informations
b. information
c. information’s
d. None of the above
Ans: Option B
Q: The punch knocked out for of his ____
a. tooth
b. teeth
c. tooths
d. None of the above
Ans: Option B

Kinds Of Noun
Now that we have learned about nouns, let us learn about certain kinds or types of nouns including the
collective nouns. Having knowledge about the various kinds of nouns will ensure we use them correctly in
a sentence. Let us get started.

Types of Noun

Proper Noun
The first kind of noun we will deal with is a proper noun. A proper noun is the name of a specific individual
person or place or thing of some kind. Basically, if the noun is a name of a unique entity we call it a proper
noun. For example, the name of a person or a city or the name of a company is all proper nouns.

So “Mumbai” is a proper noun, but “city” is just a common noun. Similarly “Jaya” is a proper noun, “girl” is
not.
One thing to remember, when dealing with a proper noun we always capitalize the first letter of the name.
It is one of the easiest ways to recognize a proper noun.

Collective Nouns
You must have heard the expression “a flock of birds”. This is a collective noun. It is a name given to a
collection of specific things. Here are some examples of collective nouns commonly used.

 A herd of animals (herbivores)


 Class of students
 Flock of birds
 Pride of lions
 Choir of singers
Uncountable Nouns
This is what we call a mass noun. These nouns refer to things that simply cannot be counted or measured.
Take for example the following sentence, “His happiness showed on his face’. Here the noun happiness
cannot be said to be counted or measured. Also “The rain came pouring down”. Can you measure this rain?
So as a general rule of thumb, we treat uncountable nouns as a singular noun and accordingly use
singular verbs with them. Some example of uncountable nouns is hair, rain, sadness, truth, cash, rice,
bread, butter etc.

Countable Nouns
The opposite of uncountable nouns, nouns which can be counted and measured are countable nouns. We
can enumerate these nouns. Examples are cats, dogs, human, pans, pots, shoes etc. Basically, it is any noun
which we can tally the number of.
Whether the countable noun is singular or plural will obviously depend on the situation. Like for example,
it can be, “A cat was sitting by the door” or “A few cats were sitting by the door”. Most of the nouns we come
across are countable nouns.

Abstract Noun
Moving on we come to the abstract noun. Abstract nouns are as the name suggests nouns that cannot be
touched or seen. These are intangible things. They can be a feeling or a thought or even a quality. Abstract
nouns are always treated as singular.
Some examples of abstract nouns are happiness, sadness, idea, courage, intelligent, evil, power etc.

Solved Example for You

Type 1
Q: Find the correct sentence: All the

a. rice fell to the ground


b. rices fell to the ground
c. rice fell to the grounds
d. rices fell to the grounds
Ans: Option “A” is correct. In the given case rice is a collective noun and we treat them as singular nouns. So
it will be rice and not rices. Also ground here may be a common noun, but it is still singular.
Q: Find the correct sentence: He needed to

a. transport the furnitures immediately


b. transports the furniture immediately
c. transport the furniture immediately
d. transports the furniture immediately
Ans: The correct option is “C”. Here furniture is an uncountable noun, so we must treat it as a singular
noun. Also, the verb to go with it must also be singular, which is “transport” and not “transports’.

Type 2
Q: Ram(1) was very upset that the barber(2) had messed up his hairs(3). Which part of the sentence is
incorrect?

a. 1
b. 2
c. 3
d. The sentence is correct
Ans: The correct answer is “C”. Hair is a collective noun, so we treat it like a singular noun. The correct
formation of the sentence is, “Ram was very upset that the barber had messed up his hair”.
Q: His experience (1) made him a better person (2) and a better teacher to his students (3). Which part of
the sentence is incorrect?
a. 1
b. 2
c. 3
d. The sentence is correct

Ans: The correct option is “A”. While experience is an abstract noun, it can be used in the plural form if the
situation demands. Here the word”experience” is a countable noun and must be used in the plural form.
The correct sentence will be “His experiences made him a better person and a better teacher to his
students.”
Q: The herd(1) of fish(2) swam towards the horizon(3). Which part of the sentence is incorrect?

a. 1
b. 2
c. 3
d. The sentence is correct
e. Ans: The correct answer is option “A”. While herd is a collective noun for animals, the collective noun
we use for fish is “School’. So the correct form of that sentence will be “The school of fish swam
towards the horizon”. Also, note it will be fish and not fishes.

Practice Questions
Q: It was a crowded city (1) the New York (2), but Alex was still enjoying the trip (3). Which part of the
sentence is incorrect?

a. 1
b. 2
c. 3
d. The sentence is correct
Ans: Option B. Proper nouns do not generally have articles in front of them.
Q: His childhood (1) was full of happiness (2) and love (3). Which part of the sentence is incorrect?

a. 1
b. 2
c. 3
d. The sentence is correct
Ans: Option D.
Q: Find the correct sentence.

a. They entered the girls college without permission.


b. They entered the girl’s college without permission
Ans: Option A
Pronouns
We have already covered the correct use of nouns, the other important topic often tested in
the banking and other competitive exams are pronouns. Let us study the types and the correct use of
pronouns in grammar.

Pronouns

A pronoun is a word that can be used in the place of a noun. We do not repeat nouns in a sentence, it would
get tedious and lengthy. So instead we use pronouns. Some examples of pronouns are she, her, he, his,
theirs, mines, yours etc.
So any word which effectively replaces a noun is a pronoun. So instead of saying “Raj went to the market,
and when Raj came back Raj was tired”, we say “Raj went to the market, and when he came back he was
tired”.

Types of Pronouns

 Personal Pronoun: These are the pronouns that replace people, i.e. names. Examples are I, we, he,
she etc. There are two types of personal pronouns, subjective (she, he, I) and objective (her, him,
me).
 Possessive Pronouns: Pronouns that represent ownership or belonging are possessive pronouns.
Examples- her, mine, his, their
 Absolute Possessive Pronouns: Similar to possessive pronouns, the difference is that they are not an
adjective to the noun. They are used individually. Example: hers, theirs, yours etc.
 Demonstrative Pronouns: Those which are used to indicate something or someone. Example: this,
that, those etc.
 Indefinite Pronouns: Opposite to the demonstrative pronouns, indefinite pronouns refer to no
particular thing or person, i.e. they talk about non-specific things. So examples will include us, them,
they, all, few etc. This is the largest subcategory of pronouns.
 Interrogative Pronouns: These are simple to spot. They are the pronouns which we generally use in
questions. Examples: Who, where, whom, how etc.
 Reflexive Pronouns: These are used to address another noun or pronoun in the sentence. They end
with -self or selves. Examples are myself, themselves, herself etc.

Rules regarding Pronouns


1] A proposition will be followed by an objective pronoun. Example: The fight was
between me and him. Here between is a preposition, so the following pronouns (me, him) will be objective
pronouns.
2] When comparing two entities and using the word “than”, nominative pronouns will be used. Example:
Alex is better than him.
3] When using a pronoun for a collective noun, the pronoun should be singular and not plural. Example:
The class is quite today. Since the class is a collective noun, pronoun will be singular.
4] If we talk about two nouns using either or/neither nor, singular pronoun will be used. Example: Neither
Sam nor Dean gave his homework on time.
5] If the nouns are joined by or, the use plural pronouns. Example: Neither Sam or Dean
gave their homework on time.

6] When using reciprocal pronouns, “each other” is used when talking about two persons. “One another ” is
used for more than two people. Example: “They knew each other from school” (talking about two people).
“Nobody knew one another on that bus” (talking about several people).

Solved Questions for You

Type 1
Q: If you ask me (1), the real contest is between you (2) and I (3). Which part of the sentence is incorrect?

a. 1
b. 2
c. 3
d. The sentence is correct
Ans: The correct option is “C’. If there are two pronouns separated by between, both should be objective
pronouns. I is a subject pronoun. The correct sentence will be ” If you ask me, the real contest is between
you and me”.
Q; My friend and myself (1) decided to visit the fair, but he (2) cancelled at the last minute (3). Which part
of the sentence is incorrect?

a. 1
b. 2
c. 3
d. The sentence is correct
Ans: The correct option is “A”. Myself is a reflexive pronoun. But in the subject of the sentence, we use a
subjective pronoun. The correct sentence will be “My friend and me decided to visit the fair but he
cancelled at the last minute”.

Type 2
Q: Which of the following sentences is incorrect.

a. “It is your mistake” he shouted.


b. “It is you’re mistake” he shouted.
Ans: Option “B” is the incorrect sentence. Your is the correct pronoun in this sentence. You’re is short for
‘you are’, not the correct grammar here.
Q: Which of the following is incorrect

a. No one liked the idea, myself included.


b. Either Jay or Neil must make his intentions known.
c. The herd of cows are not moving fast enough.
d. All are correct.
Ans: Option “C” is the incorrect sentence. A herd of cows must be followed by a singular form of the
pronoun. The correction will be as follows “The herd of cows is not moving fast enough”.

Practice Questions
Q: It (1) was a very confusing night, they (1) did not even know what to do with themself (3). Which part of
the sentence is incorrect?
Ans: Part 3 is correct. It will be ‘themselves’.
Q: “What are (1) the chances of my (2) getting through?” she (3) asked. Which part of the sentence is
incorrect?
Ans: Part 2 is correct. It will be ‘me’.
Q: Which is the correct sentence?

a. Both friends looked at each other nervously.


b. Both friends looked at one another nervously.
Ans: Option “A” is correct.

Verbs
Verbs are the action words in a sentence. They are extensively covered in banking and PO exams due to
their importance. Let us learn about the types of verbs and the correct use of verbs.

Verbs
The Verbs in a sentence tell you what the subject of a sentence is doing, or how it is being. They are
associated with people, tense, voice of speech or mood. Verbs are the heart of a sentence, they help the
sentence convey the core message. Even just the verb can express a thought, like for example”Run!” or
“sit.”
Here we will be covering some important type of verbs that are generally confusing. Once you learn how to
use the correct form of verbs, it will be smooth sailing.

Action and Non-Action Verbs

Action Verbs
Action Verbs indicate an action or the occurrence of an event. They are used when something is happening
in a sentence. Even if the word is something that happens figuratively (like a symbol) it will be an action
verb. Let us take a look at some examples.

 He ran across the street, (Ran is an action verb).


 That is her sitting on the bench (Sitting is an action verb as it implies an action).
 It was time to break the tradition and start a new trend (the break is used symbolically here, but is
still an action verb).

Non-action
Non-action verbs do not indicate an occurrence of an event, they indicate the mood, emotion,
feeling, opinion etc. Some examples of non-action verbs are
 I have the best seat in the house.
 He felt cold all of a sudden.
 I am a great player.
 She is on her way.

Phrasal Verb
Now there are many other kinds of verbs such as transitive, intransitive, auxiliary, modal etc. The one we
will be focusing on here is a special verb known as Phrasal Verb.
Here the verb is not one word, but a string of words, or a phrase. The meaning of the original word will
differ from that of the resulting phrase. We form the phrase by using a verb followed by an adverb or
a preposition. Let us look at some examples of phrasal verbs. These are just some examples, there are
hundreds upon hundreds of phrasal verbs.

Phrasal
Meaning Example
Verb

Explain or
No one can account for his
Account for justify
whereabouts on Monday.
something

Write/ take
Let me jot down my address,
jot down down
you can mail the letter to it.
something

a deal or
adjust to How do you cope with the daily
cope with
something requirements of your job.
or someone

move or
progress He has missed a few days, but he
Get on
from must get on with his studies.
something

He needs to get over the loss,


Forget and
Get over and start the business from
move on
scratch.
Verbs and Tense
It is a verb that will indicate to us the tense of a sentence. So a verb will take on a different form, in
different tenses. To ensure logic in our writing the correct form we use the verb to match the tense. Let us
take a look at an example

 I get down at the station. (simple present)


 I got down at the station. (simple past)
 I will get down at the station. (simple future)
 I am getting down at the station (simple continuous)

Solved Questions
Q: There was (1) a great number (2) of people in the street walking (3) towards the entrance. Which part
of the sentence is incorrect?

a. 1
b. 2
c. 3
d. Sentence is correct
Ans: Option “A” is the answer. People here are in the plural sense, so ‘was’ is wrong, since it refers to
singular. The correct sentence will be “There were a great number of people in the street walking towards
the entrance.”
Q: Despite their devotion and dedication (1) the business failed (2) to take in (3). Which part of the
sentence is incorrect?

a. 1
b. 2
c. 3
d. Sentence is correct
Ans: Option “C” is the answer. ‘Take in’ means to take care of someone, here the phrasal verb should be
‘take off”, which means to be successful. Therefore correct sentence here is ” Despite their devotion and
dedication the business failed to take off.”
Q: The chairman (1) and the director is (2) being investigated by the CBI (3) for fraudulent
activities. Which part of the sentence is incorrect?

a. 1
b. 2
c. 3
d. Sentence is correct
Ans: Option “B” is the correct answer. The two nouns Chairman and Director are separated by an ‘and’, but
both are preceded by ‘the’. So both are being talked about separately and so plural verb must be used, i.e.
‘are’. So the correct sentence will read as “The chairman and the director are being investigated by the CBI
for fraudulent activities.”
Practice Questions
Q: All the gold is (1) accounted for (2), I checked (3) twice. Which part of the sentence is incorrect?

a. 1
b. 2
c. 3
d. Sentence is correct
Ans: Option B. It will be ‘accounted for’.
Q: The hotel has a revolving (1) restaurant, it is (2) what made (3) it famous. Which part of the sentence is
incorrect?

a. 1
b. 2
c. 3
d. Sentence is correct
Ans; Option C. “makes’ not ‘made’.
Q: The mother as well as (1) her children are (2) out on a picnic by the river (3). Which part of the sentence
is incorrect?

a. 1
b. 2
c. 3
d. Sentence is correct
Ans: Option C. Will be ‘is’ not ‘are’.

Adjectives and Adverbs


When preparing for competitive banking exams a lot of focus must be given to adjectives and adverbs.
These two are very important parts of speech. Let us learn about adjectives and adverbs and their correct
use in English grammar.

Adjectives
An adjective is any word that describes a noun. It gives us more information about the subject of our
sentence, which is the noun. They could be describing the quality, quantity, state, shape etc of the noun.
Some examples of an adjective are big, few, beautiful, scary, tasty etc.
So an adjective will modify a noun (describe) for the benefit of the reader. Take for example the following
instance: “Alex has a dog”. This sentence will give you the information that Alex has a dog. But if we add the
appropriate adjectives, the sentence will be “Alex has a beautiful, brown, furry dog”. Now in this instance,
you have far more information about the dog. It is easy to spot the adjectives (beautiful, brown and furry)
as they are directly before the noun.

Adverbs
Now adverbs describe verbs and adjectives and also other adverbs. They modify the verbs and help us get
more information about the verb or the noun by modifying the adjective. Let us see some examples of this,

 She was hurt badly (describes the verb hurt)


 The painting was very beautiful. (describes the adjective beautiful)
 He ate his food too quickly and chocked on a morsel. (describes the other adverb quickly)
Now it is very easy to spot an adverb. Usually the end with “-ly”. But there are words where the adjective
and the adverb form of the word is same. Like for example ‘fast’. It can be both an adjective or an adverb.

Adjectives vs Adverbs
There are many ways an adjective and an adverb relate to each other. One is obviously that we use adverbs
to describe adjectives in many cases. Another relation is that people often use an adverb for linking verbs
instead of the adjective.
A linking verb is a verb which relates to feelings and other such intangible things. It describes a sensory
experience (like feeling or taste or smelling). The adjectives we use for such verbs are known as predicate
adjectives. And confusing the need of an adverb instead of a predicate adjective is a common mistake. Let
us take a look at an example,

 “The room smells badly”. Now, this may seem like a correct sentence but it is not.
 The verb smell here is a linking verb, so we will use a predicate adjective and not the adverb ‘badly’
 So the correct sentence will be “The room smells bad”.

Solved Examples
Q: There was no doubt (1) in his mind that his project (2) was the most best (3). Which part of the sentence
is incorrect?

a. 1
b. 2
c. 3
d. Sentence is correct
Ans: The correct option is C. Here the adjective ‘best’ (good, better, best) is already a superlative. It does
not need ‘most’ as an added superlative to form a compound adjective. So the correct sentence will be
“There was no doubt in his mind that his project was the best.”
Q: Rahul’s mother was unhappy (1), his marks were lowest (2) than the class average (3). Which part of the
sentence is incorrect?

a. 1
b. 2
c. 3
d. Sentence is correct
Ans: The answer is B. Here Rahul’s marks were not the lowest in the class, but they were less than the class
average marks. So the degree of comparison should be the comparative degree. The adjective ‘lower’
would be the correct form. The correct sentence will be “Rahul’s mother was unhappy, his marks were
lower than the class average”.
Q: Australia is (1) a very unique (2) country (3). Which part of the sentence is incorrect?

a. 1
b. 2
c. 3
d. Sentence is correct
Ans: The correct option is B. There are some words that do not have degrees of comparison. By their very
meaning, they convey they are superlative. Some such words are unique, universal, entire, ideal etc. So the
phrase ‘very unique’ or ‘most ideal’ etc are wrong. The correct sentence will be “Australia is a unique
country”.
Q: The students felt sadly (1) that their teacher (2) was leaving (3). Which part of the sentence is incorrect?

a. 1
b. 2
c. 3
d. Sentence is correct
Ans: The answer is A. The word felt will imply that it is a linking verb, dealing with sensory things. So we
know that instead of using an adverb (sadly in this case) we must use an adjective. So the correct sentence
will be “The students felt sad that their teacher was leaving”.

Practice Questions
Q: The surgeon (1) efficiency stitched (2) up the wound (3). Which part of the sentence is incorrect?

a. 1
b. 2
c. 3
d. Sentence is correct
Ans: B (correct adverb form is “efficiently”)
Q: He could only eat (1) the fruits as everything (2) else contained eggs and he was allergic (3). Which part
of the sentence is incorrect?

a. 1
b. 2
c. 3
d. Sentence is correct
Ans: A (could eat only; adverb will follow the verb)
Q: Don’t ride your bicycle (1) so dangerous (2) in the middle of the road (3). Which part of the sentence is
incorrect?

a. 1
b. 2
c. 3
d. Sentence is correct
Ans: B (dangerously is the correct adverb)
Paragraph Jumbles
In this section on Para Jumbles, we will focus mainly on solved examples. This section is very common in
the most prestigious exams like CAT, IBPS, SSB, RRB and many others. Hence a good understanding and
practice are warranted here. Let us see!

Para Jumbles
Most of the information given in the section on parajumbles is unnecessary for the purpose at hand, i.e.,
sorting the sentences. In essence what we are looking for are things that can help us in connecting the
sentences. Some approaches are given below to help identify the sequence of sentences. Generally, in a
given parajumble more than one approach will be applicable at the same time, therefore practices
identifying which approach/approaches apply to the parajumbles you have to solve.

Solved Examples – Type I

Noun-Pronoun Relationship Approach


In noun/pronoun relationships, we know that the noun will come first and will be referred to later using
suitable pronouns. Study the following example:
A. People can get infected by handling reptiles and then touching their mouths or an open cut.
B. At first, they look the perfect pets: exotic, quiet and tidy.
C. A study estimates that in 1995, there were as many as 6,700 reptile-caused salmonella infections.
D. But lizards and other pets can harbour a salmonella bacterium that makes people sick.
A) BCAD B) BCDA C) ACDB D) BDCA
Answer: Read sentences C and D carefully. Sentence D contains the noun phrase “a salmonella bacterium”
and Sentence C contains the noun phrase “salmonella infections”. What is the relationship between the
two? Since the phrase “a salmonella bacterium” introduces the bacterium, it should logically precede the
phrase “salmonella infections”. Therefore, the sentence that contains the phrase “a salmonella bacterium”
should come before the sentence that contains the phrase “salmonella infections”. So, Sentence D should
precede Sentence C! Once you have a link between two sentences, look at the answer choices to see if you
are on the right track. If you are, then you have the right answer [(Option (d) BDCA)].
Example 2: These enormous “rivers” – quite inconstant, sometimes shifting, often branching and eddying
in manners that defy explanation and prediction – occasionally cause disastrous results.
A. One example is El Nino, the periodic catastrophe that plagues the West Coast of America.
B. It is rich in life.
C. This coast is normally caressed by the cold, rich Humboldt Current.
D. Usually, the Humboldt hugs the shore and extends 200 to 300 miles out to sea.
5. It fosters the largest commercial fishery in the world and is the home of one of the mightiest game fish
on record, the black marlin.
A) ABCD B) DCAB C) ACDB C) CBAD
Answer: Read sentences A and C carefully. Notice the noun/ pronoun relationship between the two.
Sentence A refers to “the West Coast of America” and Sentence C talks about “this coast”. Which coast?
Obviously “the West Coast of America”! Therefore, Sentences A and C are related and Sentence A must
come before Sentence C. Now look again. Sentence C talks about “the cold, rich Humboldt Current” and
Sentence D refers to “the Humboldt” obviously these two sentences are also related. Which one should
come first? Once you have decided, check the answer choices to see if you are correct. [(Option (c) ACDB)].
Acronym Approach
Full Form vs. Short Form When we introduce someone or something, we use the complete name or title.
When we refer to the same someone or something later in the paragraph, we use just the surname or the
first name if we are on familiar terms with the person being discussed. If we are discussing an object, we
remove the modifiers and just use the noun or a pronoun to refer to it. In Parajumbles we encounter full
and short names or sometimes acronyms of some term or institution.
Example 3: A. If you are used to having your stimulation come in from outside, your mind never develops
its own habits of thinking and reflecting.
B. Marx thought that religion was the opiate because it soothed people’s pain and suffering and prevented
them from rising in rebellion.
C. If Karl Marx was alive today, he would say that television is the opiate of the people.
D. Television and similar entertainments are even more of an opiate because of their addictive tendencies.
(a) BACD (b) ADBC (c) BCDA (d) CBDA
Answer: Sentence B has Marx (short form) and sentence C has Karl Marx (full form). So, C will come before
B. Now look at the options. In option (a), (b) and (c), B is placed before C—hence, rejected. Option (d) is the
right answer.

Practice Questions
DIRECTIONS for the Questions: Sentences given in each question, when properly sequenced, form a
coherent paragraph. Each sentence is labelled with a letter. Choose the most logical order of the sentences
from among the four given choices to construct a coherent paragraph.
Q 1:
A. By reasoning, we mean the mental process of drawing an inference from two or more statements or
going from the inference to the statements, which yield that inference.
B. So logical reasoning covers those types of questions, which imply drawing an inference from the
problems.
C. Logic means if we take its original meaning, the science of valid reasoning.
D. Clearly, for understanding arguments and for drawing the inference correctly, it is necessary that we
should understand the statements first.
A) ACBD B) CABD C) ABCD D) DBCA [CAT – 1998]
Answer: B) CABD
Q 2: Commercially reared chicken can be unusually aggressive, and are often kept in darkened sheds to
prevent them from pecking at each other.
A. The birds spent far more of their time — up to a third — pecking at the inanimate objects in the pens, in
contrast to birds in other pens which spent a lot of time attacking others.
B. In low light conditions, they behave less belligerently but are more prone to ophthalmic disorders and
respiratory problems.
C. In an experiment, aggressive head-pecking was all but eliminated among birds in the enriched
environment.
D. Altering the birds’ environment, by adding bales of wood-shavings to their pens, can work wonders.
6. Bales could diminish aggressiveness and reduce injuries; they might even improve productivity since a
happy chicken is a productive chicken.
A) DCAB B) CDBA C) DBAC D) BDCA [CAT – 2000]
Ans: D) BDCA
Subjective Verb Agreement
 Verbs must agree with subjects in number and in person (1st/2nd/3rd). A subject is
a noun or pronoun. A verb is an action performed by the subject. In the following section, we will
see many such examples of the Subjective Verb concept. We also have exercises that will help you
cement the concept of Subjective Verb in a proper way.
 Subjective Verb Agreement
 Verbs must agree with subjects in number and in person (1st/2nd/3rd). Subjects and verbs must
agree in number. This is the cornerstone rule that forms the background of the concept. For
example, consider this: The dog growls when he is angry. The dogs growl when they are angry.
 Following is a roadmap or rules that you can follow to avoid any mistakes in the Subject-Verb
Agreement:

 Part I
 Don’t get confused by the words that come between the subject and verb; they do not
affect agreement. The dog, who is chewing on my jeans, is usually very good.
 Prepositional phrases between the subject and verb usually do not affect agreement. For example,
The colours of the rainbow are beautiful.
 When sentences start with “there” or “here,” the subject will always be placed after the verb, so care
needs to be taken to identify it correctly. For example, “There is a problem with the balance sheet.
Here are the papers you requested.”
 Subjects don’t always come before verbs in questions. Make sure you accurately identify the subject
before deciding on the proper verb form to use. For example:
Does Lefty usually eat grass? Where are the pieces of this puzzle.
 If two subjects are joined by and, they typically require a plural verb form. Example: The cow and
the pig are jumping over the moon.
 The verb is singular if the two subjects separated by and refer to the same person or thing. For
example: Red beans and rice is my mom’s favorite dish.

 Part II
 If the words each, every, or no come before the subject, the verb is singular. No smoking and
drinking is allowed. Every man and woman is required to check in.
 If the subjects are both singular and are connected by the words or, nor, neither/nor, either/or, and
not only/but also the verb is singular. For example: Jessica or Christian is to blame for the accident.
 The only time when the object of the preposition factors into the decision of plural or singular verb
forms is when a noun and pronoun subjects like some, half, none, more, all, etc. are followed by a
prepositional phrase. In these sentences, the object of the preposition determines the form of the
verb. For example:
All of the chicken is gone. All of the chickens are gone.
 The singular verb form is usually used for units of measurement. For example: Four quarts of oil
was required to get the car running.
 If the subjects are both plural and are connected by the words or, nor, neither/nor, either/or, and
not only/but also, the verb is plural. For example:
Dogs and cats are both available at the pound.
 Part III
 If one subject is singular and one plural and the words are connected by the words or, nor,
neither/nor, either/or, and not only/but also, you use the verb form of the subject that is nearest
the verb. For example:
Do your sisters or your girlfriend want any pizza?
 Indefinite pronouns typically take singular verbs. Example: Everybody wants to be loved. Except for
the pronouns (few, many, several, both) that always take the plural form. For example:
Few were left alive after the flood.
 If two infinitives are separated by and they take the plural form of the verb. For example:
To walk and to chew gum require great skill.
 When gerunds are used as the subject of a sentence they take the singular verb form of the verb, but
when they are linked by and they take the plural form. Example:
Standing in the water was a bad idea. Swimming in the ocean and playing drums are my hobbies.
 Collective nouns like the herd, senate, class, crowd, etc. usually take a singular verb form. Example:
The herd is stampeding.
 Titles of books, movies, novels, etc. are treated as singular and take a singular verb. For
example: The Burbs is a movie starring Tom Hanks.
 Final Rule – Remember, only the subject affects the verb!


 Prepositional Phrases and Why They Matter
 Example: The boxes of cake mix are on the shelf.
 The verb is “are,” but what is the subject? Is it “boxes” or “cake mix”? In order to figure out the
subject, we must eliminate the prepositional phrase which is often times in between the subject and
the verb.
 A prepositional phrase is a two – to four – word phrase (sometimes more) that begins with a
preposition (above, among, at, below, beneath, between, in, of, over, to, under). A verb must agree
with its subject, not with the object of a prepositional phrase, which often comes between the
subject and the verb.
 Example: The boxes of cake mix are on the shelf.
 Since “of” is a preposition, we ignore the prepositional phrase “of cake mix.” Therefore, “boxes” is
the plural subject, which matches perfectly with the plural verb “are.”
 Practice Questions:
 Q 1: Everyone (has/have) done his or her homework.
 Ans: has
 Q 2. Each of the students (is/are) responsible for doing his or her work.
 Ans: is
 Q 3. Either my father or my brothers (is/are) going to sell the car.
 Ans: are
 Q 4. Neither my sisters nor my mother (is/are) going to sell the house.
 Ans: is
 Q 5. The samples on the tray in the lab (need/needs) testing.
 Ans: need
 Q 6. Mary and John usually (plays/play) together.
 Ans: play
 Q 7. Both of the dogs (has/have) collars.
 Ans: have
 Q 8. Neither the dogs nor the cat (is/are) very hungry.
 Ans: is
 Q 9. Either the girls or the boy (walk/walks) in the evening.
 Ans: walks
 Q 10. Either the boy or the girls (walk/walks) in the evening.
 Ans: walk
 Q 11. At the end of the fall (comes/come) the hard tests.
 Ans: come
 Q 12. The slaughter of animals for their fur (has/have) caused controversy.
 Ans: has
 Q 13. The student, as well as his teacher, (was/were) going on the field trip.
 Ans: was
 Q 14. The hard tests (comes/come) at the end of the fall.
 Ans: come
 Q 15. Both of my roommates (has/have) decided to live in the dorms.
 Ans: have

Conjuctions & Prepositions

In English grammar, we often form long and complex sentences. There are a few connecting words that
help us build the bridges between two simple sentences and allow us to form fuller sentences. Let us learn
about two such connecting categories of words Conjunctions and Prepositions.

Conjunctions
Conjunctions are words that link other words to form complex sentences. A conjunction does not abide by
any particular rules, they can connect any two words together (does not have to be nouns, pronouns,
adjectives). Because conjunctions can connect two verbs, we use them to connect simple sentences.
Some common examples of conjunctions are: and, because, but, yet etc.

 She will pick up the eggs and the vegetables from the market
 He wanted to study but he could not find the time
 She was late because her car broke down.

Types of Conjunctions

Coordinating Conjunctions
These are connecting words that can join words, sentences, phrases etc that are of the same grammatical
rank (similar syntax). The most common coordinating conjunctions are and, for, but, so, or, yet, nor etc. Let
us look at some example

 She went to get food and some medicines from the grocery store
 Mr. Jones neede a vacation but he could not afford to leave work.
 There were no buses that day so I took the car

Correlative Conjunctions
These conjunctions are in pairs. They are to be written together in the same sentence. Some examples are
neither and nor, either and or, not only and but also. Some examples are

 Either we catch the next train or we will have to walk home


 Neither Adam nor his sister was at home
 Not only did he break the window, but he also refused to pay the damages

Subordinating Conjunctions
These are the conjunctions that come before and introduce the subordinating or dependent part/clause of
a sentence. They join the dependent and the independent clauses. It will usually indicate a contrast or an
effect or some such relation. Some common subordinating conjunctions are, after, before, until, though,
etc.

 I don’t want to go to class because I did not finish my assignment.


 After we get home we will start cooking dinner
 Until Alex comes home, we cannot leave the house.

Prepositions
Prepositions too are connecting words that help make sense of a sentence. They generally connect a noun
or a pronoun with the rest of the sentence. They precede nouns/pronouns in a sentence and help to form
the relation of the noun with the sentence. For example, Alex left his books at school.
Some other examples of preposition are too, with, of, between, on, until, behind etc. These prepositions at
a time, or a location or direction or some type of relation. Let us see a few examples of all.

 Time: We have not been to church since Christmas


 Direction: Once you see the post office turn to the right
 Location: Alex saw his teacher at the supermarket
 Space: The book was under his bag the whole time

Conjunctions vs Prepositions
There are some words that can be both conjunctions and prepositions. There are certain subordinating
conjunctions which are also prepositions. Some such examples are: before, after, until, since etc.
If the word is followed by a dependent or subordinating clause then it is a conjunction. If the following
words are objects, nouns or pronouns then it will be a preposition. Let us look at some examples

Here until is followed by the noun ‘sunrise’ and


Alex decided to wait until sunrise to go home
so is a preposition

Alex will wait here until we finish talking about Here until connects the subject of the sentence to
his behavior its dependent clause, so it is a conjunction

Here since is followed by a noun, so it is a


I have been having these dreams since December
preposition in this case

Here since connects the independent and the


I have been having these dreams since the
dependent clauses of the sentence. so it is a
accident
conjunction

Solved Examples
Q: Such was his handwriting (1) as I could not (2) read anything (3). Which part of the sentence is
incorrect?

a. 1
b. 2
c. 3
d. Sentence is correct
Ans: The correct option is B. The correct conjunction here will be “that”. so the correct sentence will be,
“Such was his handwriting that I could not read anything”
Q: Not only (1) was the bus driver very late (2) but he was rude (3) to the passengers. Which part of the
sentence is incorrect?

a. 1
b. 2
c. 3
d. Sentence is correct
Ans: The answer is B. Not only/but also are correlative conjunctions and must always be used together in a
sentence. In the above sentence ‘also’ is missing. The correct sentence will be “Not only was the bus driver
very late but he was also very rude to the passengers.”
Q: Hardly Alex had (1) reached his class when the teacher (2) started handing out the exam papers
(3). Which part of the sentence is incorrect?

a. 1
b. 2
c. 3
d. Sentence is correct
Ans: The correct option is A. Hardly is a conjunction that should be immediately followed by the verb. So
the correct sentence will be, “Hardly had Alex reached his class when the teacher started handing out the
exam papers.”
Q: My uncle asked (1) me to inquire if all the trains (2) were upon time (3). Which part of the sentence is
incorrect?

a. 1
b. 2
c. 3
d. Sentence is correct
Ans: The correct answer is C. Upon is a proposition of space. Here the correct preposition will be ‘on’ which
relates to time. So the correct sentence will be, “My uncle asked me to inquire if all the trains were on
time.”

Practice Questions
Q: The prisoner was (1) found guilty about (2) the robbery (3). Which part of the sentence is incorrect?

a. 1
b. 2
c. 3
d. Sentence is correct
Ans: B (for, not about)
Q: We must cultivate (1) a habit to (2) being on time since a young age (3). Which part of the sentence is
incorrect?

a. 1
b. 2
c. 3
d. Sentence is correct
Ans: B (of not to)
Q: He was neither tall or skinny (1) as per the (2) given description (3). Which part of the sentence is
incorrect?

a. 1
b. 2
c. 3
d. Sentence is correct
Ans: A (either is followed by ‘nor’ not ‘or’)
Q: Mr Jones had decided (1) to leave both his job or his home (2) and move to Chicago (3). Which part of
the sentence is incorrect?

a. 1
b. 2
c. 3
d. Sentence is correct
Ans: B (and is the correct conjunction, not or)

 Phrasal Verbs
In grammar, phrasal verbs play an important part in the English language. They can be confusing at times
since the phrase has a different meaning to the individual words. And hence it is heavily tested in bank
exams such as IBPS, SSC etc. Let us learn a few important phrasal verbs.

Phrasal Verbs
Verbs are not necessarily just one word. At times an entire phrase is considered a verb. Here the main verb
is followed by an adverb or a preposition. And the entire idiom or phrase together is known as a phrasal
verb. Let us see some examples of this.

 “See to” is a phrasal verb. The verb ‘see’ is followed by a preposition ‘to’,
 Another phrasal verb is “hang up”. Here the verb is ‘hang’ which is followed by the adverb ‘up’.

One very important thing to know about Phrasal verbs is that the meaning of the phrase or the idiom will
often be different than the meaning of the individual words. So the phrase will have an individualistic
meaning. Some examples of this are as follows

 Take the phrase “blow up”. The meaning of the phrase is “to explode”. The individual words have
different meaning and the phrase have a totally different meaning.
 “Look up” is a phrasal verb that means to search for something. Again the meaning of the phrase is
different from the meaning of the words that make up the phrase.

Some Important Phrasal Verbs


Let us now take a look at some important phrasal verbs and their meanings. These verbs we use in our day
to day communication, and it is important to be aware of them.
Phrasal Verbs Meaning Example

Ask after Enquire about I met her last night, she asked after your grandmother.

Breakthrough An achievement or success The invention of planes was a breakthrough for mankind.

To be excited and lose After his recent success, Alex got carried away and ended u
Carry away
control alienating himself from his friends.

Drop in Visit someone The doctor decided to drop in and check on Mrs Smith

Have an argument or
Fall out He and his brother had a falling out over that car.
quarrel

Escape punishment or The robber got away with only a minor fine due to the lack o
Get away with
consequences evidence against him.

Hold out Resist difficulties The army held out even in sub-zero temperatures.

The police decided to look into the matter after they wer
Look into Investigate
convinced it was a robbery.

grab or accept an
Jump at I jumped at the chance to work with Mr Ray.
opportunity with eagerness

The feud between the neighbours had gone on long enough,


Make up End a fight, Compromise
was time to make up.

Stay somewhere
Put up I put up at my aunt’s place till the storm passed.
temporarily

I knew that my parents were always going to stand by m


Standby Support or help
decision.

The board will talk over the matter and come t


Talk over Discuss
a conclusion by tomorrow.
Solved Questions for You
Q: The job was taking (1) a toll on Roger (2), he was completely burnt away (3). Which part of
the sentence is wrong?

a. 1
b. 2
c. 3
d. Sentence is correct
Ans: The correct answer is C. Burn away is not the phrasal verb to be used here. The correct verb is ‘burn
out’ which means to be exhausted and tired. So the correct sentence will be “The job was taking a toll on
Roger, he was completely burnt out.
Q: As soon as he reached (1) home he set up (2) doing the housework (3). Which part of the sentence is
wrong?

a. 1
b. 2
c. 3
d. Sentence is correct
Ans: The correct answer is B. Set up means to establish something. the correct phrasal verb here should be
‘set about’ which means to start doing some work. So the correct sentence will be “As soon as he reached
home he set about doing the housework”.
Q: It was getting late (1), so the teacher decided to wind (2) his work and leave for home (3). Which part of
the sentence is wrong?

a. 1
b. 2
c. 3
d. Sentence is correct
Ans: The answer is B. Here the verb that is correct is “wind up”. ‘wind’ means air, whereas ‘wind up’ means
to wrap up something. So the correct sentence will be “It was getting late, so the teacher decided to wind
up his work and leave for home”.
Q: As the storm weakened the wind died ____.

a. down
b. away
c. off
d. none of the above
Ans: The correct answer is A. ‘Die down’ means to gradually disappear or weaned. So the correct sentence
will be “As the storm weakened the wind died down”.

Practice Questions
Q: The heavy rains (1) caused the car (2) to break out (3). Which part of the sentence is wrong?

a. 1
b. 2
c. 3
d. Sentence is correct
Ans: C (break down)
Q: She decided (1) to look after (2) her old school friends (3) on Facebook. Which part of the sentence is
wrong?

a. 1
b. 2
c. 3
d. Sentence is correct
Ans: B (look up)
Q: He put up (1) an appearance of innocence (2) till the guilt caught up (3) with him. Which part of the
sentence is wrong?

a. 1
b. 2
c. 3
d. Sentence is correct
Ans: A (put on)
Q: Mary instantly broke ____ in tears on after the exam.

a. in
b. up
c. out
d. down
Ans; D
Q: Even though their wealth was gone, the family decided to keep ___ with appearances.

a. up
b. in
c. to
d. none of the above
Ans: A

 Articles
There are certain words in the English language that we use rather frequently. And so it is essential that
we learn their correct usage and syntax. Articles are three such words, and they are also important from
an exam point of view. Let us get started.

Articles
Articles in a way are similar to adjectives. They also define a noun. Articles will help us specify whether the
noun is a specific noun or an unspecific noun. There are two types of articles in Grammar. Let us learn
about them and their correct usage.
Indefinite Articles
There are two indefinite articles “a” and “an”. Indefinite articles are used to talk about non-specific nouns.
They refer to a noun which is generic rather than one particular thing. Take the following examples.

 Mary wanted to go to a park (This implies that mary wanted to go to any park, not a particular one)
 Could you pass me a pencil? (Here again, talk is about any pencil at hand, not one specific one)

A or An
Now let us discuss when we must use ‘an’ instead of ‘a’. The general rule is that if the following noun will
begin with a vowel (a, e, i, o, u) then we use ‘an’ and not ‘a’. For example, “She went out and bought an
umbrella.” However, there are a few exceptions to this rule. Let us take a look at them,

 If the noun starts with a consonant, but we pronounce it as a vowel, we will use ‘an’. This happens
when the first letter of the word is silent. Take for example the word ‘honest’.
 And if the word starts with a vowel, but we pronounce it with a consonant voice, then we will not use
‘an’ but use ‘a’.

Definite Article
There is only one word which we call the definite article, it is “the”. The definite article is used when we are
talking about specific nouns. The use of “the” points to the reader that the noun we are talking about is one
specific thing, person, place etc. Let us see an example,

 Mary wanted to go to the park. (here we are talking about a specific park that Mary wanted to visit.
We may have introduced the club earlier in the write up)
 Could you pass me the pencil? (Again here the pencil being talked about is a specific pencil)

Omission of Article
Now there are certain nouns before which we do not use any articles as it is not necessary. Let us see a few
such incidents where we omit the articles before nouns.

 We generally do not use articles before abstract nouns. These are intangible nouns which cannot be
counted.
 Also, we do not generally use articles before proper nouns. So we do not put a/an/the before names
of people or places etc
 Another time we generally omit articles is before plural nouns.

Solved Questions for You


Q; It was the women (1) who brought about a revolution (2) in the industry (3). Which part of
the sentence is incorrect?
a. 1
b. 2
c. 3
d. Sentence is correct
Ans: The correct option is A. Women is a plural noun, and so there is no need of an article before the plural
noun. The correct sentence will be “It was women who brought about a revolution in the industry”.
Q: There was no doubt (1) in my mind that Alex (2) was a honest man (3). Which part of the sentence is
incorrect?

a. 1
b. 2
c. 3
d. Sentence is correct
Ans: The correct option is C. Although honest starts with a consonant (H) bit we pronounce it like a vowel
(O). So we will use “an” instead of “a”. So the correct sentence will be “There was no doubt in my mind that
Alex was an honest man”.
Q: Rome is a (1) smallest country (2) in the world (3). Which part of the sentence is incorrect?

a. 1
b. 2
c. 3
d. Sentence is correct
Ans: The correct option is A. Since we are talking in superlative terms the correct article here would be
“the’. If the word was smaller than it would be ‘a’. So the correct sentence will be “Rome is the smallest
country in the world.
Q: She tried (1) one more time but (2) it was a unsuccessful (3) attempt. Which part of the sentence is
incorrect?

a. 1
b. 2
c. 3
d. Sentence is correct
Ans: The correct option is C. The noun “unsuccessful” starts with a vowel. So we use ‘an’ in place of ‘a’. The
correct sentence will be “She tried one more time but it was an unsuccessful attempt”
Q: Since bus (1) was late, we did not reach (2) the game on time (3). Which part of the sentence is
incorrect?

a. 1
b. 2
c. 3
d. Sentence is correct
Ans: The correct option is A. Bus is a common noun, and we use an article before it. Since we are talking
about a specific bus, we will use “the”. So the correct sentence will be “Since the bus was delayed we did not
reach the game on time”.

Practice Questions
Q: The official language (1) of India is the Hindi (2) and English (3). Which part of the sentence is incorrect?

a. 1
b. 2
c. 3
d. Sentence is correct
Ans: B (no article before languages)
Q: He was (1) on the terrace (2) flying the kites (3). Which part of the sentence is incorrect?

a. 1
b. 2
c. 3
d. Sentence is correct
Ans: C
Q: Mark was told (1) she will be there in a hour (2), so he rushed over (3). Which part of the sentence is
incorrect?

a. 1
b. 2
c. 3
d. Sentence is correct
Ans: B

Tense
English Grammar has many topics such as Nouns, Pronouns, Verbs, Adjectives, Modals, Tense, Active and
Passive Voice etc. Let us learn more in detail about the topic of Tense in this article. Many banking and
aptitude tests have a section on English grammar, where they ask you about the different grammar rules
and sentences related to tense, active/passive voice, modals, etc. Therefore it is imperative to be
methodical while learning the English Grammar. The article here explains the topic Tense in detail.

Tense and the types of Tense


Tense plays a very important role in the sentence structure. It tells us more about the time at which the
action has occurred or will occur or is occurring. When we speak of tense, it is the tense of a verb that we
are speaking about. This tense tells you more about what the subject of the sentence has done or will do or
will be doing. In the English Language, there are three main tenses. They are the Present Tense, Past Tense
and the Future Tense. Furthermore, there are four forms of each of these different types of tense.
Types of Tenses – Source – Pinterest
The following table shows the different forms of tense and their usage and structure.

Present tense

Tense Usage Structure

It is used to express a
Subject + Verb (base
Simple Present habitual action, fact or
form)+ Object
general truth.

Used to describe an
Is/am/are + ‘ing’ form
Present Continuous action going on at the
of present participle
time of speaking.

To express an action
Has/have + past
Present Perfect that has just got
participle
completed.

Used to express an
Has/have+ been+ ‘ing’
Present Perfect action that began in the
form of the present
Continuous past and is still
participle
continuing.

Past Tense

Tense Usage Structure

Simple Past Generally used to Subject + past participle


express a regular action + object
that has occurred in the
past.

Expresses a continuing
was/were + ‘ing’form of
Past Continuous action at some point in
present participle
time in the past.

It is utilised to express
an action that was
Past Perfect has + past participle
completed at a certain
time in the past.

Used to express an
action which began
had + been + ‘ing’ form
Past Perfect Continuous before the time of
of present participle
speaking and continued
up to that time.

Future Tense

Tense Usage Structure

To express an action
Shall/Will + base form of
Simple Future that will take place in
the verb
the future

This is used to express


an action that will be in Will/Shall be + ‘ing’ form
Future Continuous
progress at a given point of present participle
in time in future.

This tense is used to


express an action which
Will/shall have + past
Future Perfect will be completed by a
participle
certain time in the
future.

Future Perfect It is used to express an Will/ shall have been +


Continuous action that will have ‘ing’ form of present
been going on at or participle
before some point in
time in the future.

Solved Questions
Check out the following examples and questions for a better understanding.
Fill in the blanks with the right form of the tense of the verbs given in the bracket.

1. Rani sleeps for more than ten hours. ( Sleep – Simple Present Tense)
2. He will pass the examination this year. ( Pass – Simple Future Tense)
3. We were playing cricket yesterday. ( Play – Past Continuous)
4. The postman is delivering letters. ( Deliver – Present Continuous)
5. He felt hot and sticky. ( Feel – Simple Past)
6. He will be reading a novel at this time tomorrow. ( Read- Future Continuous)
7. All the students have paid their fees. ( Pay – Present Perfect)
8. The train had arrived at the station when I reached there. ( Arrive – Past Perfect)
9. She will have done her work before I reach. ( Do – Future Perfect)
10. India has been progressing very fast since independence. ( Progress – Present Perfect Continuous)
11. I had been waiting for you since morning. ( Wait – Past Perfect Continuous)
12. Tomorrow, at this time, I shall have been flying to London for two hours. ( Fly – Future Perfect
Continuous)

More Practice Questions


Fill in the blanks with the right form of the tense.

1. She often …… to the Cinema because she loves movies.

A. is going
B. go
C. going
D. goes
Ans. Option D
2. We ….. to a new place, as the old place was too small for us.

A. shifted
B. shifts
C. is going to shift
D. shifts
Ans. Option A
3. Ram … football in the same club as me.

A. plays
B. played
C. play
D. will have played
Ans. Option A

 Active and Passive Voice


As you are already aware, there are many rules that are followed in the English Language, especially
grammar. Right from the sentence structure, tenses, voice, usage of prepositions, nouns etc. there are
many rules that have to be followed. These rules are quite simple, even though at first they may appear
complicated. When you follow these rules properly, you can master the English Grammar tricks with ease.
In IBPS exams and many competitive exams, you will come across many questions from English Grammar.
Among these, there are questions related to active and passive voice. Let us look at this topic in detail.

Grammar Rules for Active and Passive Voice


A transitive verb has two forms or two voices. These are the Active and passive.
Active Voice – Here, the subject performs the action. He/she is the doer of the action. It is a pretty
straightforward relationship between the subject and the verb. So, we can say that a verb is in the active
voice when the subject is the doer of the action that is expressed by the verb.
The sentences in Active Voice are simple and easy to understand.
Look at the following examples:

 I am drinking tea.
 He played cricket.
 They will help you.
See the structure of Active voice and Passive voice in more detail here.
Passive Voice – Here, the subject receives the action that is expressed by the verb. Therefore, we can say
that the verb is in passive when the subject of the verb is acted upon.
It is generally used to show the action, which means that the focus is on the action and not the subject who
does the action. Sentences in Passive Voice are not simple, as more importance is given to the action rather
than the subject.
Take a look at the following examples:

 Tea is being drunk by me.


 Cricket was played by him.
 You will be helped by them.
Source – Pinterest

Changing the voice from Active to Passive


When a verb changes from Active Voice to Passive, the subject and object change places with each other.
The past participle form of the verb is used as the main verb in passive voice.
Read the rules for changing from Active voice to Passive voice in more detail here.
Look at the following table:
Tense Active Passive
Simple present keep is kept
Present continuous is keeping is being kept
Simple past kept was kept
Past continuous was keeping was being kept
Present perfect have kept have been kept
Past perfect had kept had been kept
Simple Future will keep will be kept
Conditional Present would keep would be kept
Conditional Past would have kept would have been kept
Present Infinitive to keep to be kept
Perfect Infinitive to have kept to have been kept
Present Participle keeping being kept
Perfect Participle having kept having been kept

You Might Also Like This Article:

 Figure of Speech
 Punctuation

Solved Questions

Change the sentences from Active Voice to Passive Voice

1. Active Voice – She does not cook food.


Passive Voice – The food is not cooked by her.

2. Active Voice – Peter gave me flowers on my birthday.


Passive Voice – I was given flowers by Peter on my birthday.

3. Active Voice – You are waiting for your friend.


Passive Voice – Your friend is being waited for by you.

4. Active Voice – The children have broken the window pane.


Passive Voice – The window pane has been broken by the children.

5. Active Voice – I shall have my car sold.


Passive Voice – My car will have been sold by me.
More Practice Questions
In the following questions, change the Voice into Passive Voice. Choose the right option.

1. The boy laughed at the beggar.

A. The beggar laughs at the boy.


B. The beggar was being laughed by the boy.
C. The beggar was being laughed at by the boy.
D. The beggar was laughed at by the boy.
Ans. Option C
2. Stella will invite Rita.

A. Rita will be invited by Stella.


B. Rita will have been invited by Stella.
C. Rita will want to be invited by Stella.
D. Rita will be wanting to be invited by Stella.
Ans. Option A

Direct/Indirect Speech
In exams, there are questions where you to convert the indirect speech into direct speech and vice versa.
Thus, students cannot afford to skip direct/indirect speech or make any mistakes in it. Because in the
exam you have to attempt all the questions.

Direct/Indirect Speech

Rules for converting direct/indirect Speech


Following are the golden rule to convert the direct/indirect speech

 Step 1: Make a note of the reporting verb that is used to decide the tense of the indirect speech.
 Step 2: Change the place and time to indicate the actual place and time that is meant by the speaker.
 Step 3: Also, use an appropriate pronoun for the object and the subject.
 Step 4: Additionally, make a proper structure/word order of the sentence.
Now, we will discuss each of these steps in detail

Step 1: Deciding the conversion and tense of the verb


Case 1: Ram said, ‘I go to the class every day’.
Case 2: Ram says, ‘I go to the class every day.’
In both the cases above, ‘say’ is the verb that conveys the action of speaking. Also, in the 1st case, the
reporting verb say is given in the past tense – said. While in case 2, the reporting verb is given in the
present tense.
So, here the rule is for all verbs to be in the corresponding past tense. Also, this is followed if the reporting
verb is in the past tense. Thus, Ram said, ‘I go to the class every day’ will change to Ram said that he went to
the cleverly day.
The 2nd rule here is that if the reporting verb is in the future or present tense then the tense is not
changed. So, Ram says, ‘I go to the class every day’ will be changed to Ram says that he goes to the class
every day.

Step 2: Changes are done in the word expressing Place, Time, and Connection
Place or time mentioned in the sentence should be changed accordingly to indicate the actual time or
place.
On 1st, April 2015: ‘I will come tomorrow,’ he said.
On 2nd June 2015: He said that he would come the next day.
‘She was not here,’ Ramya said. She said that she was not there.

Step 3: Selection of pronoun for the object and the Subject


Case 1: He will say to his friends, “I have started learning arts.” => He will tell his friends that he has started
learning arts.
In this, the speaker along with the reporter is the same. So, the pronoun should be the first person which is
a pronoun.
Case 2: The professor said to me, “I hope you will bring the book to my class.” => The professor hoped that I
would bring the book to her class.
Here, the professor is the speaker while the student is the reporter. So, the pronoun of the professor
should be in thirx people. Also, the pronoun of the reporter should be in the first person.

Practice Questions on Direct/Indirect Speech


Q. Rina said, “I am not doing well in the exams.”
A. Rina said that she is doing well in the exams.
B. Rina said that she was not doing well in the exams.
C. Rina said that she has not done well in the exams.
D. None of the above
Answer: B. Rina said that she was not doing well in the exams.

English Grammar Practice Questions


English Grammar Practice Questions
English Grammar Practice Questions will contain questions on the various topics of grammar. The section
on English Grammar Practice Questions has questions on Noun and its kinds, Pronouns, Verbs, Adjectives,
Adverbs, Paragraph Jumbles and many more such topics. Here in the English Grammar Practice Questions,
we will see many questions on these topics and others that we have learned in the English grammar
section. Let us see more!

English Grammar Practice Questions

Type I
In these questions, an idiom is given by four alternatives. The candidate is required to choose that
alternative which correctly expresses the meaning of the given idiom.
Directions: In each of the following questions, an idiomatic expression/ a proverb has been given, followed
by some alternatives. Choose the one which best expresses the meaning of the given idiom/ proverb.
Q1: To turn over a new leaf [Railways, 1995]
A) To change completely one’s course of action. B) To shift attention to new problems after having
studied the old ones thoroughly. C) To cover up one’s fault by wearing new marks. D) To change
the old habits and adopt new ones.
Q2: A fair crack of the whip
A) Severe punishment B) A good luck check C) A period of importance D) Failure of
administration
Q3: To talk one’s head off
A) To talk loudly B) Talk in whispers C) To talk to oneself D) Talk excessively
Q4: To wrangle over an ass’s shadow [SBI PO 1984]
A) Act in a foolish way B) To quarrel over trifles C) Waste time on petty things D) To do
something funny.
Q5: To take with a grain of salt
A) Take with some reservation B) To take with total disbelief C) Take wholeheartedly D) To
take seriously.
Q6: Hobson’s choice [Railways, 1991]
A) Feeling of insecurity B) Accept or leave the offer C) Feeling of strength D) Excellent
choice
Q7: To take the bull by the horns [Railways, 1995]
A) To punish a person severely for his arrogance. B) To grapple courageously with a difficulty that
lies in our way. C) To handle it by a fierce attack. D) To bypass the legal process and take action
according to one’s own whims.

Find Your Answers Here


Q1: (d), Q2: (c), Q3: (d), Q4: (b), Q5: (a), Q6: (b), Q7: (b)

Type II
Directions: I each of the following questions, four or five alternatives are given for the phrase in italics in
the sentence. Choose the one which best expresses the meaning of the phrase in italics.
Q1: He has come up during the last five years. [Stenographer’s Exam, 1991]
A) fallen in rank B) risen in status C) become proud D) become humble
Q2: After tolerating a lot of exploitation at the hands of the authorities they finally decided to stand up for
their rights.
A) seize B) enforce C) negotiate D) vindicate
Q3: The boss should not have called Arun names in front of others.
A) abused B) summoned C) scolded D) praised
Q4: Last evening I was held up at the meeting. [NDA 1995]
A) kept B) detained C) stopped D) delayed
Q5: I have a problem to square up with the manager.
A) work out B) consider C) discuss D) settle
Q6: Sachin is not cut out for this kind of work.
A) work out B) consider C) discuss D) settle

Find Your Answers Here


Q1: (b), Q2: (d), Q3: (a), Q4: (b), Q5: (d), Q6: (c)

Type III
Directions: In each of the following questions, a sentence has been given in Active (or Passive) Voice. Out of
the four alternatives suggested selecting the one which best expresses the same sentence in Passive (or
Active) voice.
Q1: Mona was writing a letter to her father.
A) A letter was written to her father by Mona.
B) A letter has been written to her father by Mona.
C) A letter was being written by Mona to her father.
D) A letter was written by Mona to her father. [Stenographer’s Exam 1994]
Q2: He teaches us Grammar.
A) Grammar is taught to us by him.
B) We are being taught Grammar by him.
C) grammar is being taught to us by him.
D) We are taught Grammar by him.
Q3: Do you imitate others?
A) Are others imitated by you?
B) Are others being imitated by you?
C) Were others being imitated by you?
D) Have others been imitated by you? [Stenographer’s Exam, 1995]
Q4: I saw him conducting the rehearsal.
A) He was seen conducting the rehearsal.
B) I saw the rehearsal to be conducted by him.
C) He was seen by me to conduct the rehearsal.
D) I saw the rehearsal being conducted by him.
Q5: Have the box broken. [Clerk’s Grade, 1996]
A) Have the broken box. B) Break the box. C) Get someone to break the box. D) They have
broken the box.

Find Your Answers Here


Q1: C), Q2: D), Q3: A), Q4: D), Q5: C).

Reading Comprehension:

Paragraph Fillers
Paragraph fillers test the reading comprehension of the candidates. A paragraph of three or more lines is
given here. Following the paragraph are four sentences that will either sum up the paragraph or make it
meaningful in one way or the other. The sentences summarise or steer the paragraph in a given direction.
Let us see with the help of the following sections. The basic thought here is that a paragraph contains one
coherent thought.

End Paragraph Fillers


Paragraph Fillers are a new section in the banking exams. Let us see the first kind of examples where the
paragraph fillers go at the end.
Example 1: In the medieval times, the average lifespan of the human beings was way lesser than what it is
now. The advancements in the modern medicine and other technological advancements have made it
possible. _________________________.

1. Polio has been almost eradicated by the improvements in science.


2. In medieval times, the science of medicine was not yet developed.
3. As with more and more advancements in science, the average lifespan of humans keeps increasing.
4. Science is very important for a long and healthy life.
Answer: As more and more advancements in science are made, the average lifespan of humans will keep
increasing. Well, this is an obvious choice as the paragraph talks about medieval times, then the present
and naturally will have to move into the future. Also, all the other options are less appropriate as the
ending of the paragraph.

Example 2: Galileo was persecuted by the Roman Catholic Church, to the point that he was put under house
arrest and his books were banned. When the great physicist, Dr Stephen Hawkings was invited to the
Vatican, it is said that the Pope asked him what did he wish to see? The Physicist answered, “ The trial
records of Galileo”. __________________.

1. Stephen Hawking is thus a great fan of Galileo.


2. The Pope was irritated and asked the great physicist to leave.
3. The Pope asked Dr Hawking to kiss his ring first.
4. This points to the conflict between science and religion.
Answer: The first option could be an answer but let us see if there are any better options to pick. The
second sentence is not coherent with the train of thoughts in the paragraph. The third sentence is also not
much relevant. However, the fourth option seems to sum up the idea presented in the paragraph. The idea
that science and religion are incoherent. Thus the answer is 4.

Random Fillers
If the sentence has to go at the end then it is most probable that it will sum up the idea presented in the
main paragraph. It should not contradict it as that would make it a part of a different paragraph. However,
the sentences that go in the middle are asked too. Let us see an example:
Example 3: Kashmir is one of the most beautiful places on Earth. It has something for everyone.
_______________________. The white snow-capped mountains, the crystal clear and cool springs, the red chinar
trees, the meadows filled with flowers, and fruit-bearing tree orchards are a sight to behold! One should
visit this paradise on earth, at least once in his lifetime.

1. There are different seasons and every season has its attractions.
2. The place is very beautiful and most safe place to travel to.
3. The people are hospitable and friendly.
4. The travel costs very less.
Answer: Let us begin considering each sentence in the options. But first, let us see what the paragraph is
talking about. The paragraph talks about Kashmir and how it has “something for everyone”. This is the
central idea presented in the paragraph and the option that you choose must be coherent with this. Now
let us analyze the options one by one.
The first option seems correct. It builds on the idea presented in the main paragraph. The second sentence,
is correct but doesn’t contribute to the idea of the paragraph and is not to be considered. Similarly, the
third and the fourth sentence are both correct but don’t seem to flow with the central idea of the
paragraph. Hence the first option is correct.

Practice Questions
Q 1: Complete the following:
First, it spins a web in the shape of a bell. Then it fixes it to plants under the water. Next, it swims up to the
top of the water and traps a tiny bubble of air with its hairy back legs. It drags the bubble to its web. And
does this many times until its home is full of air.___________________.

1. It prepares its web to inject a potent toxin into its prey.


2. Some spiders don’t spin a web.
3. Some spiders can even trap small birds in their webs.
4. Then it sits nice and snug in its web and feeds on tiny fish and other water animals.
Ans: 4) Then it sits nice and snug in its web and feeds on tiny fish and other water animals.
Q 2: She opened a Christmas package given her by friends before she sailed. ____________. The boat was
making only a half a knot, so she ran the engine for a while, and this improved her spirits. She decided to
run it a little every day while the calm lasted. It kept the engine from corroding, and the sound was good
for her morale.

1. The greetings, cards, and small gifts not only failed to cheer her but also made her feel more lonely.
2. The twin staysails were working again now, and she was creeping along on her course, but the
fatigue was beginning to tell.
3. The next day she got the wind she had asked for – too much of it.
4. A squall struck before daylight, heeling Felicity Ann dangerously.
Answer: 1) The greetings, cards, and small gifts not only failed to cheer her but also made her feel more
lonely.

Paragraph Sentence sequence


Paragraph Sentence Sequence questions ask the candidate to rearrange a group of sentences to form a
coherent paragraph. These sequence questions can be of a few types. Let us try and see some examples
and methods to solve these.

Paragraph Sentence Sequence


Let us begin with a simple example. Find the correct sequence of the following sentences:
1. In spite of an unprecedented boom in the market.
A) and had to remain content
B) the paper dealer could not
C) push up his sales
D) with the volume of sale lower than
2. what he had sold the previous year.

A. BCAD
B. DBAC
C. ABCD
D. CABD
Answer: Let us first understand the pattern of the question. The question has two fixed clauses marked
with 1 and 2. All the other sentences are to be fit between the two clauses. The trick is to look for
the pronouns. Make the combinations in your head and read them. Try and see which combinations make
sense out of the ones that you tried. In the above case, the combination BCAD seems to fit the best. The
sentence is thus “In spite of an unprecedented boom in the market the paper dealer could not push up his
sales and had to remain content with the volume of sale lower than what he had sold the previous year.”

May 20, 2012 during the Annular Eclipse of the Sun

Level II Difficulty
Let us try and increase the difficulty a bit. Remember there are no fixed rules to solve these questions. The
more you read, the better you will get. Assemble the following paragraph:

1. Scientists have learned to supplement the sense of sight in numerous ways.

A. Or they look through a small pair of lenses arranged as a microscope into a drop of water
or blood and magnify by as much as 2000 diameters the living creatures there, many of which are
among man’s most dangerous enemies.
B. In front of the tiny pupil of the eye they put, on Mount Palomar, a great monocle 200 inches in
diameter, and with it see 2000 times farther into the depths of space.
C. Or they can bring happenings of long ago and far away as coloured motion pictures, by arranging
silver atoms and colour-absorbing molecules to force light waves into the patterns of original reality.
D. Or, if we want to see distant happenings on earth, they use some of the previously
wasted electromagnetic waves to carry television images which they re-create as light by whipping
tiny crystals on a screen with electrons in a vacuum.

2. Or if we want to see into the centre of a steel casting or the chest of an injured child, they send the
information on a beam of penetrating short-wave X rays, and then convert it back into images we can
see on a screen or photograph. Thus almost every type of electromagnetic radiation yet discovered
has been used to extend our sense of sight in some way.
Answer: The correct sequence is BADC. If you follow the ‘or’s you will see the sequence is very natural.

Practice Questions
Q 1: Find the correct sequence of the sentences in the paragraph:

1. Rowson and Moor (1966) conducted the study of the development of sheep conceptus during the
first fourteen days.

A. By Day 16 to 18, it extended into the contralateral horn.


B. Eckstein and Kelly (1977) reported that during early pregnancy in ewes, the trophoblast rapidly
elongated to occupy both horns and body of the uterus by Day 20.
C. They found that by Day 13 to 14 the embryonic vesicle lied as a 10 cm long tube in the uterine horn
ipsilateral to the corpus luteum of pregnancy.

2. King et al (1982) found that on Day 11 trophoblast vesicle began to elongate, by Day 13th vesicle
entered the contralateral horn, by Day 18 both horns were occupied in ewes.
Ans: C, A, B.

Level II Difficulty
Q 2: Detect the correct sequence of the sentences in the following:

1. Finally, he found his car keys in the back of his right pocket of his blue-jeans.

A. Everything he had dreamed of, everything he had hoped for, all his life raced in his head in jumbled
blurry images when he felt the cold steel of a gun pointed at him and a hoarse male voice telling him
to give him his wallet.
B. Somehow, he knew this wouldn’t end well. He just couldn’t have that perfect life he had dreamed of,
could he?
C. Then things took a turn for the worse.
Ans: C, A, B.
Q 3: Suggest the best sequence of the sentences that result in a coherent paragraph:
Long ago, in a land of mist and magic, a land forgotten by time itself, in a small village, a county of the
kingdom there lived a
A. young boy with big ambitions: he dreamed of becoming a knight in the king’s army, King Harold’s
army.
B. His parents were old and needed him there to help them with the animals and with the work around
the house.
C. Mark, the young village boy, was an only child.
D. He was sure that life meant more than that. It had to. Village life was not for him.
E. But he wanted more than that. He didn’t want to spend the rest of his life caring for the pigs and
goats.

2. He was aware that his parents needed him but on the other hand, he couldn’t bury himself there.
Ans: A, C, B, E, D

Paragraph Based Questions


Paragraph Based questions include questions that ask you to complete, rearrange and
present conclusions of a given paragraph. They are part of the reading comprehension section of the
banking exams and are very important with respect to the scoring. Normally you will encounter questions
that ask you to select an option that sums up or presents the ideas that are in a paragraph. We will see
these and many examples in the following sections.

Paragraph Contents
In these type of questions, you spot certain things or points that have been discussed in a given paragraph.
For example, let us see the following paragraph:
Example 1

Read the following passage and answer the questions at the end.
And yet it is one of the simplest ideas that anyone ever had. Here I want to persuade you
how evolution explains the beginning of life on earth. Darwin uncovered the theory of evolution and the
method of natural selection. The idea of evolution is probably one of the most important ideas that anyone
has ever had. Today, thanks to Darwin, we know why life is the way it is. We can predict how life will be in
the future. We can even postulate about the life on other planets. How amazing is that! Now answer the
following questions:

Q 1. What is the topic of the paragraph?


A) The idea of evolution is simple.
B) Evolution is important.
C) It tells us about the origin of life.
D) By evolution, we can make predictions.
Answer: The answer is C.
Although by evolution, we can make predictions is also mentioned in the paragraph but it is not the central
idea of the paragraph. The central idea is usually at the beginning or you will find a few sentences that are
focussed on the idea.
Example 2

Read the paragraph and answer the question that follows:


Sedimentary rocks are formed by the process of sedimentation. Layer after layer of minerals is deposited
over a great span of time, resulting in the formation of a sedimentary rock. As a result, each layer is
different if the conditions under which its deposits were different. Thus we can say that a sedimentary
rock is a sort of museum, holding the records of all the time over which it was formed, which by all means
can be as long as a billion years.
Q 1. A sedimentary rock gets its name from the fact that
A) These rocks are formed up of small sediments of minerals.
B) It is very old and is a slow process
C) Sedimentary rocks are formed as a result of sedimentation of minerals.
D) The rocks are broken into small sediments easily.
Answer: The answer, if you read the paragraph carefully, you will have guessed is C. Note that sometimes a
paragraph may contain information that is not correct, but you will have to select the answer that is
present in the paragraph, not the one that you think is factually correct.

Paragraph Data comprehension


Sometimes, the given paragraph will be a collection of facts or data that you will have to use to be able to
answer the questions. Let us see an example.
Shoaib, Harshad, Preksha, and Yawer are four friends playing poker. In each hand, the individual player
has to bet 800 rupees. Preksha wins three hands, Yawer wins two and the others win four hands each. Now
answer the following questions:
Q 1. The person who lost the most money is
A) Shoaib
B) Harshad
C) Preksha
D) Yawer
Answer: The person who wins the least hands is the person who lost the most money. As every one of the
four friends plays an equal number of hands. In this case, Yawer wins the least and is the person who has
lost the most money.
Q 2. How many hands were in total?
A) 7
B) 2
C) 3
D) 8
Answer: Preksha wins 3, Yawer 2 and the others 4 each.

Practice Questions
Q 1. Read the following passage and answer the questions that follow.
The constitution of India is very well written. It not only defines the need for fundamental human rights
but also ensures that we do not abuse these laws. The access to justice is one such law but in the present
political scenario, this right like other such rights has been hidden from the marginalized section of the
society. As a result, we can say that for such classes of the society, there is no such thing as a constitution.
The above paragraph says what about the Indian constitution?

A. It is very well written.


B. Could be better.
C. It ensures everyone has basic rights.
D. The marginalized sections of the society don’t enjoy the rights guaranteed by the constitution.
Answer: D is the correct option.
Q 2. Read the following paragraph and answer the question that follows:
Space exploration is a very exciting field of research. It is the frontier of Physics and no doubt will change
the understanding of science. However, it does come at a cost. A normal space shuttle costs about 1.5
billion dollars to make. The annual budget of NASA, which is a premier space exploring organization is
about $17 billion. So the question that some people ask is that whether it is worth it?
What is the main idea in the paragraph?

A. NASA has a budget of $17 billion.


B. The cost of making one space shuttle can be used for other constructive work.
C. The money spent on NASA can be used otherwise.
D. Space exploration is costly and some people are worried.
Answer: D is the correct option.

Paragraph Conclusions
Paragraph conclusions or paragraph inference questions forms a section of the reading
comprehension.The main aim of this section is to check your ability to understand and read a given
paragraph in a short span of time. These questions will present a paragraph and a set of sentences in the
form of options. Out of these options, you will have to select the one that best describes the ideas present
in the given paragraph.

Paragraph Conclusion Tips


Although reading comprehension will only get better and better if you practice more and more, you may
be mindful of a few points while attempting this section. First of all, never overthink or draw conclusions.
For example, consider the following paragraph:
There were many street dogs on her way to the school. They were all very vicious and had been known in
the past to have bitten children of her age. When she returned home with a bite mark on her calf, it was
clear to her mother what had happened!

Which of the following most accurately represents the ideas presented in the paragraph?
A) The girl was afraid of dogs.
B) The dogs were very timid.
C) The girl was bitten by one of the stray dogs.
D) The girl was bitten.
Answer: Well, you say, the answer is clearly C. The girl was bitten by one of the stray dogs. You would be
wrong. Read the paragraph again. It doesn’t say it anywhere that the girl was bitten by a stray dog. Out of
the four options given, the only option that is actually mentioned in the paragraph is that the girl was
bitten. Hence, the answer is D.

To Conclude or Not To Conclude


Only draw conclusions when it is beyond doubt that the thought that you have concluded follows all cases
from the paragraph. For example consider the same paragraph, only this time the options will be different.
Which of the following most accurately represents the ideas presented in the paragraph?
A) The girl was alone on the streets.
B)) The dogs chased the girl and bit her.
C) The girl was bitten by one of the stray dogs.
D) The girl’s mother was very angry at her.
Answer: So which one is it now? Let us examine each option carefully. Option A suggests that the girl was
alone on the streets which may be true or not. The paragraph simply doesn’t talk about it. Also, there is no
clue or reason to believe it. So A can’t be the answer. Now let’s move on to B which says that the dogs
chased the girl and bit her. Again, in the given paragraph, although one might infer that the girl might have
been bitten by one of the dogs, it will be wrong to infer that they chased her first. This information is just
not presented in the paragraph and shouldn’t be inferred from it.
Let us observe option C saying the girl was bitten by one of the stray dogs. This can be inferred from the
paragraph. But we won’t select it yet unless we see every option. So let us move on to the last option D
which again is something that can’t be concluded from the clues given in the paragraph. So the right option
is C.

Practice Questions
Q 1. Read the paragraph carefully and answer the questions at the end.
What if I told you that the moon was once present right here on the surface of the earth! Well not exactly in
the form that you see now, but for many years there was the hypothesis that in a destructive event of
gigantic proportions, the moon was actually part of the earth that just separated from it. maybe the earth
collided with another ancient planet that doesn’t exist anymore or a huge meteor or comet just snapped
part of the earth after it collided with it. It is possible but is that what actually happened. Many scientists
argue against it and propose a different hypothesis that the moon was formed by the debris leftover after
the formation of the earth.
According to the author, how was the moon formed?

A. It originated from the earth.


B. It was always there.
C. From the debris of an ancient collision between earth and a big heavenly object.
D. God made it.
Ans: C is the correct option.
Q 2. Read the following passage and answer the questions at the end.
The Bible, many believe is the word of God. But scientists that include many reputed Physicists and
Biologists refute that claim. In fact, science refutes the claims of almost all religions as being divine. The
superstitions and miracles of the religious are equally ridiculed by science via the scientific community. In
the words of a great Physicist, Steven Weinberg, it is corrosive of religion.
What is meant by the phrase “It is corrosive of religion”?

A. That the scientific community is corrosive to religion.


B. Science is corrosive of religion.
C. The scientific community and science are corrosive of religion.
D. Superstitions are corrosive of religion.
Ans: B is the correct option.

Paragraph Restatement
 In IBPS exams there are mainly two types of paragraph restatement questions. In the preliminary
exam, you will have to read a passage and answer many questions that will ask you questions on the
main ideas present in the paragraph. However, in mains, your writing skills are checked and you are
asked to write a paragraph precis. This article should help you answer both types of questions
comfortably.
 Paragraph Restatement
 In the following section, a passage is present. At the end of the passage, questions are given. All you
need to do is select the correct option in each case.

 Passage I
 The author dedicates this work to the thousands of sympathetic and well-wishing friends of the
Negro race. He is trying to show how the Negro problem can be solved in peace and goodwill rather
than by brutality. His idea is that the Golden Rule furnishes the only solution. He believes that at the
bottom of southern society there is a vein of sympathy and helpfulness for the Negro and that this
feeling should be cultivated and nourished that it may grow stronger and finally supplant harsher
sentiments.
 There are two factions striving for the mastery of the south to-day, one seeking political power on
the idea that Negro manhood is to be crushed and serfdom established, and the other willing that the
Negro should have a freeman’s chance and work out his destiny as best he can with the powers God
has given him. This faction is ready to give its sympathy and help, and it is the efforts of this class that
the author desires to endorse and encourage.

 The story weaved into the work is subordinate to the discussion of facts, and not paramount; it is
intended to be mild, thus putting it in keeping with the character of the heroine whose deeds it
portrays; and should the day ever come when America can rise to the height of adopting and
following her sentiments, it will then indeed be the “Sweet land of liberty,” for the black as well as the
white man.

 Questions Based On The Passage


 Q 1: The passage is about a
 A) Book
 B) Person
 C) Work of art
 D) Race
 Answer: The passage begins with the mention of an author. If you read the passage further, it
mentions the trails and tribulations of a particular race. The passage also talks about a book. At this
point, you will notice that almost all the options that are present, find a mention in the paragraph.
But the main topic of the passage is the one in the topic sentence i.e. it is a passage from a book.
Hence the answer is A) Book.
 Q 2: Of the following options, which one will you say best describes the author of the passage?
 A) The author of this passage believes that African-Americans were always treated equally in the
American society.
 B) African-American people were well represented in the American society since the beginning of
the civil rights moment.
 C) America needs to adopt a social setup that is similar to sentiments of the heroine of the play to be
truly free and fair.
 D) America needs to declare its society free for all and establish an equality before the law for all its
citizens, irrespective of colour and creed.
 Answer: Although a few options are logically correct, they are not the correct option here. By this,
even option D) is fair, but it is not mentioned in the passage and thus can’t be the correct option.
Similarly, option A) and B) are factually incorrect as nothing of this sort is mentioned by the author
in the passage. So the correct option is C) as this is the only one that is directly mentioned in the
passage.
 Remeber when restating or summing up a passage, never add your own views, no matter how
obvious they are.
 Practice Questions
 Read the following passage and answer the following questions.
 I am sure in the toils. Last night the Count asked me in the suavest tones to write three letters, one
saying that my work here was nearly done, and that I should start at home within a few days, another
that I was starting on the next morning from the time of the letter, and the third that I had left the
castle and arrived at Bistritz. I would fain have rebelled, but felt that in the present state of things it
would be madness to quarrel openly with the Count whilst I am so absolutely in his power; to refuse
would be to excite his suspicion and to arouse his anger.
 He knows that I know too much and that I must not live, lest I be dangerous to him; my only chance is
to prolong my opportunities. Something may occur which will give me a chance to escape. I saw in his
eyes something of that gathering wrath which was manifest when he hurled that fair woman from
him.
 He explained to me that posts were few and uncertain and that my writing now would ensure ease of
mind to my friends; and he assured me with so much impressiveness that he would countermand the
later letters, which would be held over at Bistritz until due time in case chance would admit of my
prolonging my stay, that to oppose him would have been to create new suspicion. I, therefore,
pretended to fall in with his views and asked him what dates I should put on the letters.

 Questions
 Q 1: Choose the incorrect statement.
 A) The Count is holding the author against their will.
 B) The author has been asked to write three letters but, the author is still considering options to
escape.
 C) The author wants to convince us that Count is a nice person irrespective of his vices.
 D) The author is being held in Bistritz.
 Ans: A) The Count is holding the author against their will.
 Q 2: Which of the words would best describe the count.
 A) Evil B) Heinous C) Murderer D) Mysterious
 Ans: D) Mysterious.

Cloze Test
 A Cloze test is an exercise where the candidate is provided with a paragraph. The given paragraph
has missing words or blanks which need to be filled. Sometimes the blanks will have a word in
the brackets accompanying it. The candidate will be asked to pick a word similar to the word in the
braces. Here we will see all types of Cloze test questions. Cloze test is a very important section and
questions are duly expected from it.

 Cloze Test
 The first type of questions in this test will include long passages that will have missing blanks and
you will have to fill these blanks with the help of words that are present at the end. Each option will
have a number or a symbol or an alphabet that will mark it. Following the paragraph passage, the
questions corresponding to the blanks will be present. Let us see with the help of an example.
 In the following passage, there are blanks, each of which has a number. These numbers are present
below the passage and against each, we have five options. One of which either replaces the blank
appropriately or they will ‘require no change’. Find out the appropriate word that should be fit in the
blank. In case, there is no correction, mark option(e) as your answer.

 Type I
 Example 1
 I do remember it. And I’m grateful. But I can’t help feeling that a woman capable of taking other
people’s lives and juggling with them as if they were India-rubber balls as she did with ours, is likely
at any moment to break out in a new place. My gratitude to her is the sort of (1) ______ [hat tip] you
would feel toward a cyclone if you were walking home late for dinner and it caught you up and
deposited you on your doorstep. Your Aunt Lora is a human (2) _____[storm]. No, on the whole, she’s
more like an earthquake. She has a habit of splitting up and altering the face of the world whenever
she feels like it, and I’m too well satisfied with my world at present to (3) _____[enjoy] the idea of
having it changed.”
 Little by little the garrison of the studio had been (4) _______ [small slice] down. Except for Steve, the
community had no regular members outside the family itself. Hank was generally out of town. Bailey
paid one more visit, then seemed to consider that he could now absent himself altogether. And the
members of Kirk’s bachelor circle stayed away to a man.
 Q 1.
 A) Tippy
 B) Gratitude
 C) Excellence.
 D) Battle.
 Answer: Gratitude
 Q 2.
 A) Cyclone
 B) giant
 C) Chin
 D) Hunter
 Answer: Cyclone
 Q 3.
 A) Good
 B) relish
 C) delicious
 D) Happy
 Answer: relish
 Q 4.
 A) slice
 B) whittled
 C) Cut
 D) levelled
 Answer: whittled

 Type II
 Complete the following paragraph with the help of the words given at the end.
 His ideas about girls and (1)[female]_______ were of a sentimental and modest quality; they were
creatures, he thought, either too bad for a modern (2) [word stock] _______, and then frequently most
undesirably desirable, or too pure and good for life. He made this simple classification of a large and
various sex to the exclusion of all (3) [halfway] _______ kinds; he held that the two classes had to be
kept apart even in thought and remote from one another. Women are made like the potter’s vessels–
either for worship or contumely and are withal (4) [soft] _____ vessels. He had never wanted
daughters. Each time a daughter had been born to him he had concealed his chagrin with great
tenderness and effusion from his wife and had sworn unwontedly and with passionate sincerity in
the bathroom.
 Q 1.
 A) birds
 B) women
 C) students
 D) nice people
 Answer: B) women
 Q 2.
 A) Diction
 B) linguistics
 C) Word well
 D) vocabulary
 Answer: D) vocabulary.
 Q3
 A) intermediate
 B) middle
 C) half
 D) split
 Q4
 A) broken
 B) damaged
 C) Clear
 D) fragile
 Answer: Fragile
 Practice Questions
 Read the following paragraph and complete it using the words given in the options:
 He was a manly man, free from any strong (1) [mother] ________ strain, and he had loved his dark-
eyed, dainty bright-coloured, and active little wife with a real vein of passion in his sentiment. But he
had always felt (he had never allowed himself to think of it) that the promptitude of their family was
a little (2) [insensitive]_______ of her, and in a sense an intrusion. He had, however, planned brilliant
careers for his two sons, and, with a certain human amount of warping and delay, they were
pursuing these. One was in the Indian Civil Service and one in the rapidly developing motor business.
The daughters, he had hoped, would be their mother’s care.
 Q 1.
 A) maternal
 B) Paternal
 C) Mother
 D) Childish
 Ans: A) maternal
 Q 2.
 A) tactful
 B) indelicate
 C) insensitive
 D) non-proportionate

Fillers
 In the IBPS and SBI eams, Fillers are a fairly new addition. We have already seen paragraph fillers
where we saw paragraphs that were missing a line from them. Here we will see similar questions, we
will fill out missing lines, words and more. Let us begin.
 Fillers
 Let us first see an example of a paragraph that takes a filler word in each sentence. We will call this
the type I examples. Let us see below.

 Type I
 In each of the questions below, the paragraphs are missing some words. Choose the words from the
given options that best fit the blanks in the paragraphs.
 Example 1: “No, no, he found it. Its owner is unknown. I beg that you will look upon it not as a
battered billycock but as an __________ (1) problem. And, first, as to how it came here. It arrived upon
Christmas morning, in company with a good fat goose, which is, I have no doubt, _______ (2) at this
moment in front of Peterson’s fire. The facts are these: about four o’clock on Christmas morning,
Peterson, who, as you know, is a very honest fellow, was returning from some small jollification and
was making his way homeward down Tottenham Court Road.”
 A) intellectual (1), roasting (2)
 B) serious (1), dancing (2)
 C) intelligent (1), cooking (2)
 D) unpleasant (1), pecking (2)
 Answer: Let us see the first filler. When reading the sentence, we see that there is a hint word “but”
after battered billycock. This means that whatever goes in the blanks must be opposite in meaning to
battered billycock. Also, the use of the article “an” rules out the option B). A billycock means a British
feather hat, a battered billycock thus means a hat that has been overused or damaged. This phrase is
for something that is unpleasant. This rules out option D).
 Now out of the two options, C) and A) the blank (1) could be filled with either of the two. Thus we will
look at the second filler. For the second filler, we see the goose is put on the fire. Thus it is
roasting on the fire. Hence the correct filler is A).

 Type II
 In the questions given below, a paragraph has a missing sentence. The missing sentence is present
below in the options. Select the correct option.
 Example 2: In the 1960’s the government committees set out to change the condition of the
marginalised sections of the society. As a result, numerous affirmative action schemes were
announced. ___________________. Perhaps the problems are too intricate to be solved by a scheme. We
need to change the way people think and behave.
 A) These schemes were very successful.
 B) These schemes worked on a very limited level.
 C) It is a shame that none of it has been able to abolish the marginalisation from the society.
 D) The schemes were not made for the rich and the affluent but for the poor and the needy.
 Answer: The answer is C) It is a shame that none of it has been able to abolish the marginalisation
from the society. Let us see how. Before the blank, we see that a number of schemes were announced.
After the blank, we get a picture that these schemes didn’t work and we need to do something more.
Thus we cancel A) and D) are immediate. Also, the option B0 talks of a limited scope of action while
as our paragraph states that the schemes failed entirely. So the correct option is C).
 Practice Questions
 In the following questions, either a sentence or a word is missing. Pick out the correct fillers given at
the end.
 Q 1: “The further points, that he is middle-aged, that his hair is grizzled, that it has been ______(1), and
that he uses lime-cream, are all to be gathered from a close examination of the lower part of the
lining. The lens discloses a large number of hair-ends, clean cut by the scissors of the barber. They all
appear to be adhesive, and there is a distinct odour of lime-cream. This dust, you will observe, is not
the gritty, gray dust of the street but the fluffy brown dust of the ________(2), showing that it has been
hung up indoors most of the time, while the marks of moisture upon the inside are proof positive
that the wearer _______ (3) very freely, and could, therefore, hardly be in the best of training.”
 A) (1) recently cut, (2) house, (3) perspired
 B) (1) burnt, (2) factory backyard, (3) moved
 C) (1) scratched, (2) road, (3) danced
 D) (1) trimmed, (2) river, (3) cried.
 Ans: A) (1) recently cut, (2) house, (3) perspired
 Q 2: Then the toys in the store of Horatio Mugg began to have lots of (1) _____. They told stories, sang
songs, made up riddles for one another to guess and played tag and hide-and-go-seek. They were
allowed to do all this because it was (2)_____ and no one was watching them. But as soon as daylight
came and Mr Mugg or Miss Angelina or Miss Geraldine or any of the customers came into the store,
the toys must be very still and (3) _____.
 A) (1) sweets, (2) school, (3) excited.
 B) (1) food, (2) good, (3) sleepy.
 C) (1) fun, (2) night, (3) quiet.
 D) (1) sleep, (2) day, (3) calm.
 Ans: C) (1) fun, (2) night, (3) quiet.
Inferences
 In inferences section of the bank exams, the questions will ask you to sum up or select the meaning
of a sentence, word inside a sentence or an entire passage. The idea behind the inference section is
to present the candidate with passages, sentences or phrases that have a specific meaning. These
questions check the candidate on his reading comprehension abilities.
 Inferences
 The strategy of answering inference is very simple. A candidate has to have a three-prong approach
to such questions. The first method is the deduction. In such questions, the option will be something
that is not directly visible in the passage or the sentence. Rather you will have to deduce it. Let us see
this strategy and others with some examples.

 Deduction
 Example 1: The scholarship was difficult to get. His marks were very satisfactory. He had waited
three months for the email to come. It was supposed to be the happiest day in his life. He shut
his computer and left the room.

 Q 1: After reading the above, what do you think has happened?


 A) The person got accepted into a University.
 B) The person got rejected by the University.
 C) The person is waiting for a reply.
 D) The person shuts his computer down.
 Answer: The answer is C) The person is waiting for a reply from the University. The paragraph
carries a negative sense and it makes it clear that the person was checking his computer for mail. But
it doesn’t make it clear that he has received a rejection letter, in which case he is still waiting for a
reply to come.
 Example 2: He managed to get onto the top of the hill with difficulty. The river was right below. He
readied his rifle and took aim. Below, there were deer, unsuspecting and grazing.
 Q 1: Why did the person climb on top of the mountain?
 A) He liked the view from the top.
 B) The mountaintop was a calm and relaxing place.
 C) He wanted to get a clear shot of the deer.
 D) The rifle was on top of the mountain.
 Answer: The sentence “the river was right below” seems to point to the fact that it was a good place
to take aim. The other options seem to have no backing in the passage. So the correct option here will
be B) The mountaintop was a calm and relaxing place.

 Speculation
 The word speculation means the forming of a theory or conjecture without firm evidence. In these
types of questions, you will be asked to speculate on the basis of the passage. Let us see an example
of speculation. Read the following passage and answer the question that follows.
 Example 3: All the pain and suffering comes from the stupidity of humans. Their stupidity to believe
nonsense and force this nonsense on others is what causes pain and suffering in the world. In the
words of the great philosopher called Voltaire,”If you believe in absurdities, you can commit
atrocities”.
 Q 1: What do you think the author will feel about a person who believes in ghosts?
 A) The person is absurd and should change his beliefs.
 B) The person’s beliefs are dangerous to others.
 C) The author does not say anything about ghosts.
 D) Voltaire was a great philosopher.
 Answer: You may immediately go ahead and select D) but Voltaire is not the main topic of the
passage. Please consider all options before you answer. The option A) doesn’t follow from the
passage. The passage presents the idea that a person who believes in ridiculous things or
superstitious or illogical things can easily commit evil deeds against others. So the obvious option to
infer from that is B) The person’s beliefs are dangerous to others.

 Examination
 The examination questions will present a passage and ask the candidate about the state of thought of
a given character. In other words, a passage will present the views and opinions of a particular
author and you will have to guess on those bases the feelings of the author. Let us see an example.
 Example 4: A wonderful fact to reflect upon, that every human creature is constituted to be that
profound secret and mystery to every other. A solemn consideration, when I enter a great city by
night, that every one of those darkly clustered houses encloses its own secret; that every room in
every one of them encloses its own secret; that every beating heart in the hundreds of thousands of
breasts there, is, in some of its imaginings, a secret to the heart nearest it!
 Q 1: What does the author mean by “in some of its imaginings”:
 A) That even the people who are very close keep secrets from each other.
 B) Each person has a different imagination than the other.
 C) That each person’s secrets are different than the other person that he is sharing a space with.
 D) Every person has an independent existence no matter how close he gets to others.
 Answer: The word imaginings means thoughts or fantasies. Thus the author speculates about the fact
that even though the people might seem close to each other, they can still harbour great secrets from
each other. Thus the correct option is A).
 Practice Question
 Q 1: Something of the awfulness, even of Death itself, is referable to this. No more can I turn the
leaves of this dear book that I loved, and vainly hope in time to read it all. No more can I look into the
depths of this unfathomable water, wherein, as momentary lights glanced into it, I have had glimpses
of buried treasure and other things submerged.
 In the above, what would describe the condition of the author best?
 A) The author is deeply sad and disturbed and unable to focus.
 B) The author is distracted from a book by something else.
 C) The author is depressed and wishes to read a book.
 D) The author is talking about life and how it is a book that we all have to read.
 Ans: B) The author is distracted from a book by something else.

Jumbled paragraphs
 Jumbled paragraphs are part of the reading comprehension in the IBPS PO, SO and SBI exams. The
questions on Jumbled paragraphs will have a set of sentences that all belong to the same paragraph.
The sentences are just not in the order that they should be. There will be hints all spread within the
question. Your job is to pick these hints and select the correct order of the sentences.

 Jumbled Paragraphs
 As we shall see, this section is similar to the other reading comprehension sections. But before we
start with an example let us know about the various kinds of sentences in a paragraph.

 A paragraph starts with what is known as the topic sentence. This sentence introduces the topic and
the domain of the paragraph. Then follow the supporting sentences. The supporting sentences
contain further information about the topic sentence. The conclusion or the concluding sentence is
the last part of the paragraph. It sums up the supporting sentences and also points to the topic
sentence.
 The paragraph thus has to follow a natural order: Topic sentence > Supporting Sentences >
Concluding Sentence. Your task will primarily be to spot these sentences. Let us practice with some
examples now.

 Examples of Jumbled Paragraphs


 Following are a few examples that we will be answering the Jumbled Paragraphs. In the questions,
sentences from I to IV are present. These sentences are all part of a paragraph but do not necessarily
fall in the order that they are in. Find the correct order from the options that are given at the end.
 Example 1:
 I: The main thing that propelled the development of the aeroplanes at such a fast pace was, however,
the first and the second world war.
 II: An aeroplane was a wonderful invention of its time.
 III: But its use as a main means of transport was adopted later only done later.
 IV: Some even say that it was one of the greatest inventions of all time.
 The correct order of the sentences is:
 A) I – II – IV – III
 B) II – IV – III – I
 C) III – IV – II – I
 D) IV – II – III – I
 Answer: The correct option is B) II – IV – III – I. As you can see the topic sentence introduces the
aeroplanes as the greatest invention. Now will follow the topic sentences which are IV and II. The I
sentence summarises the thought presented in the paragraph.
 Some More Examples
 Example 2:
 I: An indefinable suspicion, which he could not explain, made him uneasy.
 II: It was this, probably, that prompted him to go to the closet in which he knew that Nicholas Bundy
kept a pistol.
 III: This ought to have quieted him for the night, but it did not.
 IV: At times he placed the pistol under his pillow, but he had not done so to-night, considering it quite
unnecessary in a quiet boarding-house.
 A) II – III – I IV
 B) III – II – I – IV
 C) III – I – II – IV
 D) IV – II – I – III
 Answer: Here we shall try to find the topic sentence first. The topic sentence is the one that sets the
scene here. This is the III sentence. The second sentence will be the one that immediately follows this
thought. This is the I sentence. The rest is easy. The pistol is the word that you have to look for. So the
correct order is III-I-II-IV or the option C).
 Thus we see that there are two types of questions in the paragraph jumbling. One is which follows
the regular conclusive order. These are usually easier and all you need to do is read a sentence and
see if there is anything that is introduced without explanation. For example, a sentence like this
“these vehicles were then ordered to be redesigned” can’t be the leading sentence. It
has pronouns and terms that are yet to be introduced. A context is missing. So as long as the context
is missing, the sentence can’t be the topic sentence.
 Practice Questions
 Q 1: The following sentences are not in order. The correct order is provided in one of the options.
Select the correct option from the following options:
 I: The explanation of Mr Kenyon’s letter is briefly this.
 II: If pushed to extremity he would turn against Kenyon, and make public the conspiracy in which he
had joined, together with Kenyon’s motive in imprisoning his wife.
 III: He had had an interview with Dr Fox, in which he had so severely censured the doctor that the
latter finally became angry and defiant, and intimated.
 IV: His visit South had done no good.
 A) I – III – IV – II
 B) II – IV – III – I
 C) IV – III – II – I
 D) I – IV – III – II
 Ans: D) I – IV – III – II
 Q 2: The following sentences are not in order. The correct order is provided in one of the options.
Select the correct option from the following options:
 IV: It dislodged the green spectacles from Denton’s nose, and for a moment his eyes were exposed.
 II: To Oliver it was fun, but Denton evidently did not relish it.
 V: He replaced them hurriedly, but not in time. Oliver’s sharp eyes detected him.
 I: The road was a bad one, jolting the vehicle without mercy.
 III: At last one jolt came, nearly overturning the conveyance.
 A) IV – II – V – III – II
 B) III – IV – V – II – I
 C) IV – II – V – I – III
 D) V – IV – II – I – III
 Ans: C) IV – II – V – I – III

Paragraph Completion
When preparing for competitive exams such as IBPS and other banking exams, students should pay special
attention to the area of Reading Comprehension. Not only is it important from an exam point of view, but
also enhances your grip on the language and its understanding in general. Let us now learning the finer
aspects and some tips that will help you with paragraph completion.

Paragraph Completion
Just as we have seen sentence completion or sentence fillers, paragraph completion is of similar nature.
Here you have to pick the best option (from the given options) that perfectly fits into the paragraph
provided. So in nature, it is very similar to sentence fillers. And our technique for tackling such questions is
similar as well.
Generally, the sentence which finishes and completes the paragraph comes during the last part of the
paragraph (i.e. the concluding sentence). However, this is not always the case. There can be three types of
sentences for paragraph completion. These are,

 Initiating Sentence: Where you have to figure out what the starting sentence of the paragraph must
be. A perfect introductory statement.
 Connecting Sentence: A sentence that acts as a connector between the initiating and concluding
sentences.
 Concluding Sentence: This is the most difficult one. Your choice of the last sentence must perfectly
pair with the rest of the paragraph and conclude it in a logical and complete manner.

Tips for Paragraph Completion

1] Concept of the Passage


The first step in paragraph completion is to understand the concept and scope of the passage. So you must
figure out what the passage talks about and to what extent.
So if any of the options are outside the scope of the passage, they are eliminated from being the answer.
Such a process of elimination will ultimately make your choice simpler.
Say for example the passage talks about climate change in India. And one of the options talks about the
economic conditions of the USA. Then you know that this option is unrelated to the paragraph.
2] Tone of the Paragraph
The tone of a paragraph is represented by the views and opinions of the author. By reading the paragraph
carefully, we can determine the mood of the author towards the topic in question.
The tone can be objective in nature too, where the author does not choose sides. Hence the tone will be
neutral in such a case.
But in certain cases, the author will take a more subjective approach. And by paying attention to the
adjectives and phrases we can easily figure out if the tone of the author is negative or positive. This will
help us in making the selection of the best choice for paragraph completion.

3] Direction of the Paragraph


When reading the passage, keep a lookout for events or transitional sentences. These will dictate the
direction the passage is taking. Also, try to purview the logical reasoning behind the direction of the
passage. This will help you in eliminating some of the options that seem to take a diverse direction.
For example, if the passage is about the ill effects of global warming, and one of the passage suggests
promoting plastic usage, you can easily eliminate this option.

Practice Question on Paragraph Completion


Q: Chose the most appropriate option for paragraph completion,
India, ever since independence has been a labor-intensive economy. It has been a factor for many
industries, the advantage India provides cheap skilled and unskilled labor. While this is an initial
advantage, it is not an enduring advantage. For one, as the products get more sophisticated, the price of
labor gets less important a factor. Then, the increasing level of automation narrows the cost advantage of
cheap labor. And any poorer country can provide cheaper labor to compete with India. Thus, to remain
competitive, India needs to, __________

a. Ensure to maintain its advantage by constantly lowering labor prices


b. Try to avoid competition with countries providing cheaper labor
c. develop other advantages to compensate for the inevitable loss from a temporary advantage
d. keep on its path without worry
Ans: The correct answer is C. It is the only option that matches the tone and the direction of the passage. It
concludes the paragraph logically and in agreement with the sentiments of the author.

Double Fillers
While preparing for your banking competitive exams and even for your day-to-day life, mastering the
English language has huge benefits. One such important factor in this process is improving and honing
your vocabulary. Even for solving tricky double fillers questions this is the key. Let us take a look.

Double Fillers
Fillers are nothing but sentence fillers, an exercise in completing the sentence correctly. These fillers and
double fillers test the student’s vocabulary and grasp of the English language. This is done to ensure that
the candidates have a basic knowledge of the language and its grammatical syntax.
Single filers are when only one word or phrase is needed to complete the sentence. Double fillers, as the
name suggests, require two words to logically complete the sentence.
The candidate must choose the most appropriate option from the list of options given. Considering that
there are two important words/phrases missing, this exercise has a higher level of difficulty and
complexity.
There are certain pointers you can keep in mind while answering these tricky double fillers questions. Let
us take a look at a few of them,

1] Read the Options Last


This is one of the most important pointers for solving double fillers questions. Students in their hurry and
impatience tend to read the options/choices of the question before the question itself. This is a mistake.
It will only confuse the students more. Instead, very carefully read the sentence first. Try and understand
what the author wishes to relay. Take cues from the grammar of the sentence. Only then analyze your
choices.

2] Take Clues from the Grammar


Sometimes, you will be able to take clues from the sentence and its grammatical formation or syntax. For
example, if the blank is preceded with an ‘an’ instead of an ‘a’, this is a clue that the word begins with a
vowel.

3] Use Connotation
When we use words in a sentence we must be mindful of their connotation. Other than the definition of the
word, they also invoke a certain feeling or an idea which can be positive, negative or neutral. So in case of
questions with double fillers, look out for certain words that suggest any such idea.
For example, the word “childish” has a negative connotation. However, if the word used is “youthful’ then
the suggested connotation is positive.

4] Look out for Indicators


Sometimes there are certain words/phrases that indicate where the sentence is headed, i.e. what the
author wishes to convey. Some common ones to keep in mind are,

 Contrast Indicators: They help contrast or differentiate things. So we look for an option that has an
opposite meaning than the one used in the sentence. Some common examples are but, although, yet,
while, rather, though, etc.
 Support Indicators: Words that continue the same line of thought, i.e. support what is said.
Examples: and, also, likewise, similarly, etc.
 Cause and Effect Indicators: These words indicate how one thing led to the other, i.e. the cause and
subsequent effect of something. Examples are because, for, therefore, consequently, etc

Question on Double Fillers


Q: The student was fairly _____ of passing, yet previous experiences with this teacher left him _____.

a. uncertain, confused
b. certain, confident
c. confident, uncertain
d. confident, certain
Ans: Note the word “yet” in the sentence. This is a contrast indicator. So the words but be opposite of each
other, which leaves us with option C.

Practice Problems On Reading Comprehension


 Reading Comprehension Practice Questions section will test your knowledge on the most important
concept of the English language section. Reading comprehension is the most featured section of
many exams so we have compiled a diverse section on this concept. In the following section, you will
find many practice question that will allow you to get a complete recap as well as let you ascertain
your level.
 Reading Comprehension Practice Questions

 Type I
 Without water …….(1)……. animal can survive. In desert regions, the greatest …..(2)…. to life is dying
up. But many creatures are able to make use of …..(3)…… little water that exists in arid areas. One of
nature’s masterpieces …(4)…. creatures equipped to …..(5)….. with desert life is the hardy camels.
Stories range the desert lands far and wide about ….(6)….. endurance feats by camels. It is said that
camels can …..(7)….. a distance of about 800 miles in eight days through continuous travel …..(8)…..
an intake of a single drop of water. The popular ….(9)…. that camels store water in their hump is
….(10)…. in a way, water is indeed stored there but in the form of fat.
 1. (a) every (b) no (c) any (d) desert (e) exceptional
 2. (a) need (b) worry (c) threat (d) requirement (e) inadequacy
 3. (a) what (b) the (c) very (d) that (e) extremely
 4. (a) for (b) about (c) among (d) with (e) of
 5. (a) live (b) resist (c) bear (d) cope (e) mix
 6. (a) remarkable (b) little (c) tolerable (d) popular (e) obvious
 7. (a) measure (b) reduce (c) lead (d) reach (e) cover
 8. (a) for (b) without (c) accepting (d) receiving (e) except
 9. (a) proverb (b) guess (c) belief (d) version (e) statement
 10. (a) baseless (b) wrong (c) misleading (d) correct (e) untruthful

 Find Your Answers Here


 1. (b), 2. (c), 3. (b), 4. (c), 5. (d), 6. (a), 7. (e), 8. (b), 9. (c), 10. (c)
 Type II
 Directions: In each of the following questions, an incomplete statement followed by some fillers is
given. Pick out the best one which can complete the incomplete statement correctly and
meaningfully.
 Q1: He tames animals because he ………. (Bank P.O. 1995)
 (a) seldom loves them. (b) is afraid of them. (c) is fond of them (d) hates them (e)
wants to set them free.
 Q2: My mother is so poor …………. (S.S.C. 1993)
 (a) to send me to school. (b) because she will not work. (c) to get medical help for my
father. (d) that she cannot buy food for us.
 Q3: “Can you lend me a pencil, please?” “…………”
 (a) Yes, you can (b) Yes, here you! (c) Yes, here (d) Yes, here it is!
 Q4: Johnny, where are you? ………, up this tree. (Section Officers’ 1993)
 (a) Here I am. (b) Here am I. (c) There I am. (d) There am I.
 Q5: I don’t know the time ……….
 (a) at which the accident happened
 (b) the accident happened
 (c) when the accident happened
 (d) when the accident had happened.
 Q6: How much a man earns is as important as …………… [B.S.R.B. 1996]
 (a) when does he do so
 (b) how does he do it
 (c) where does he earn of all
 (e) how well he spends it
 Q7: One of you must help me, ………..
 (a) Shan’t you? (b) wouldn’t you?
 (c) won’t you? (d) mustn’t you?
 Q8: He is so lazy that he ………… [Bank P.O. 1995]
 (a) always extends help to others to complete their work.
 (b) dislikes to postpone the work that he undertakes to do.
 (c) can seldom complete his work on time.
 (d) can’t delay the schedule of completing the work.
 (e) can’t depend on others for getting his work done.

 Find Your Answers Here


 Q1: (c), Q2: (d), Q3: (d), Q4: (a), Q5: (b), Q6: (b), Q7: (d), Q8: (c)
 Type III
 Directions: In each question below, a theme is partially presented in two or three sentences. The
complete theme comprises three, four or five logically arranged sentences. Out of these, the middle
one or two sentences are not given. Three possible fillers denoted by A, B and C are given below the
partially provided theme. You have to find out which one or two or either or none can fill up the gap
between the given sentences in order to make the theme complete. Study the five alternatives (a),
(b), (c), (d) and (e) carefully and select the best out of them.
 Q1: In cultivating team spirit, one should not forget the importance of discipline. (………..). It is the
duty of all the members of the team to observe discipline in its proper perspective. [Bank P.O.
1994]
 A) A proper team spirit can seldom be based on discipline.
 B) It is a well-known fact that team spirit and discipline can never go hand in hand.
 C) Discipline in its right perspective would mean sacrificing self to some extent.
 (a) A and B only (b) B and C only
 (c) C only (d) Either A or B only (e) None of these.
 Q2: By reading books even our natural talents are also developed because they get inspiration from
these (…………). bacon has rightly said that some books are to be tasted, others to be swallowed and
some few to be chewed and digested.
 A) Reading is an intellectual food for the mind.
 B) There is no doubt that different persons choose different books according to their taste.
 C) Good and inspiring books can lead us to a noble and successful career in life.
 (a) A and B only (b) B and C only
 (c) A and C only (d) A or B and C (e) None of these.

 Find your Answer Here


 Q1: (c), Q2: (c).

Correct sentence:-

Error Detection
Error Detection forms a subsection of the reading comprehension. It forms a subset of the section on
correct sentences. The main aim of error detection is to be able to identify sentences that can be
corrected or that need correction. That section will follow but here in this article, we will see how to
identify wrong sentences. Let us begin.

 Error Detection
 A sentence is defined as a collection of words that have a meaning. In the English language, the
sentences usually follow a grammatical structure of subject followed by the verb and then the
object. It is also the SVO – structure. Further, there are other parts of a sentence, parts like the
articles, the tense, the punctuation, the articles, modals etc. We will see examples of each case in the
following sections. Let us begin now.

 Adverb, Article, Adjective Errors


 We shall begin with examples.
 Q 1: People (1) are believing/ (2) in the medicine that/ the government dispensaries (3) have.
 A) (1) B) (2) C) (3) D) (1) & (2)
 Answer: In these questions, the sentences have been split into sections with the help of slashes. One
or more of these sections may be incorrect. You need to identify the section and pick the number for
the section. The correct sentence can be written as: People believe in the medicine that the
government dispensaries have. So the correct option is A) (1).
 Q 2: (1) She walking (2) briskly (3) meant that she was in a hurry.
 A) (1) B) (2) C) (3) D) (2) & (3)
 Answer: The correct sentence will be: Her walking briskly meant that she was in a hurry. Thus the
correct option is (1).
 Q 3: The vehicle was not (1) moving swiftly/ as it (2) stopped frequently/ at (3) regularly intervals.
 A) (1) B) (2) C) (3) D) (2) & (3)
 Answer: If you read the sentence you will find that the noun intervals is being qualified by
the adverb regularly. An adverb qualifies a verb and not a noun. To qualify a noun we use an
adjective. The adjective of regularly is regular. Thus the correct sentence here is: The vehicle was
not moving swiftly as it stopped frequently at regular intervals. The correct option thus is C) (3).

 Other Errors
 Q 1: The (1) radically polarised/ (2) sections of the society /are (3) no good to anyone./ (4) No
error.
 A) (1) B) (2) C) (3) D) (4)
 Answer: The sentence seems to be correct. So the option to be chosen here is D (4).
 Q 2: The (1) function was held/ at Mumbai /(2) and as a result, /(3) not much of them/ (4) were able
to attend it.
 A) (1) B) (2) C) (3) D) (4)
 Answer: The phrase not much of them is used for uncountable nouns. Hence the error here is in (3)
and the correct option is C) (3).
 Q 3: (1) The then/ (2) Prime Minister had issued/ (3) an order though /he was not (4) supposedly
to/.
 A) (1) B) (2) C) (3) D) (4)
 Answer: The correct sentence is: The then prime minister had issued an order though he was not
supposed to. So the answer is D) (4).

 Passage Errors
 Sometimes the sentences will be given as part of a passage and you will be asked to pick the
incorrect sentence. Let us see with the help of a few examples.
 In the following passage, sentences have been numbered in order. Some or all of them are incorrect.
Detect the incorrect sentences.
 (1)Very orderly and methodical he looked, with a hand on each knee, and a loud watch ticking a
sonorous sermon under his flapped waist-coat, as though it pitted its gravity and longevity against
the levity and evanescence of the brisk fire. (2) He had a good leg, and was a little vain of it, for his
brown stockings fitted sleek and close, and were of a fine texture; his shoes and buckles, too, though
plain, were trim. (3) He wore an odd little sleek crisp flaxen wig, setting very close to his head:
which wig, it is to be presumably, was made of hair, but which looked far more as though it were
spun from filaments of silk or glass.
 A) (1) B) (2) C) (3) D) (1) & (3)
 Answer: The correct sentence is: He wore an odd little sleek crisp flaxen wig, setting very close to
his head: which wig, it is to be presumed, was made of hair, but which looked far more as though it
were spun from filaments of silk or glass. Thus the correct option is C) (3)
 Practice Question
 In the following passage select the incorrect sentence.
 (1) His linen, though not of a fineness in accordance with his stockings, was as white as the tops of
the waves that broke upon the neighbouring beach, or the specks of sail that glint in the sunlight far
at sea. (2) A face habitual suppressed and quieted, was still lighted up under the quaint wig by a pair
of moist bright eyes that it must have cost their owner, in years gone by, some pains to drill to the
composed and reserved expression of Tellson’s Bank. (3) He had a healthy colour in his cheeks, and
his face, though lined, bore few traces of anxiety. (4) But, perhaps the confidential bachelor clerks in
Tellson’s Bank were principally occupied with the cares of other people; and perhaps second-hand
cares, like second-hand clothes, come easily off and on.
 A) (1) B) (2) C) (3) D) (1) & (2)
 Ans: D) (1) & (2)

Sentence Improvement

Sentence Improvement section is a frequent part of the IBPS PO, SO, SBI and RBI exams. In these questions,
the knowledge about the rules of the grammar is checked. The candidate must expect questions that check
the concepts of tenses, verbs, suppositories, adjectives, modals, conjunctions, prepositions, articles etc.
The questions are present in a multiple choice format. Here we will see many examples forming the
various kinds of questions that are present in this section of the paper.

Sentence Improvement
In the paper, you will see that a sentence is present with a word or phrase or a clause in bold or
underlined. Below this option will be present. You will be asked to select the option that can replace the
word/ phrase/ clause in a way that the sentence becomes free of any grammatical mistakes. Sometimes,
however, the sentence may be given as such and you shall be asked to identify the error. Let us see this
with the help of some examples and their solved explanations.

Solved Examples -Type I


Let us see the actual text of the question that is present in the paper.
Directions: “In the following questions, a phrase/clause/word/some part of the sentence is underlined.
Which of the options present below the sentence would you choose to replace the part that is in bold? The
option that you select will make the sentence grammatically correct. If there is nothing wrong with the
sentence and it is absolutely correct grammatically, then choose the option E which is ‘No Correction
required’ as the answer.
Example 1: Besides her friends, her wife and colleagues also gets the invitation.
A) Too Gets B) Also getting C) Also got D) too getting E) No correction required
Answer: The sentence has a simple past tense, except the part of the sentence in the bold letters. Thus we
have to replace the bold out part of the option that is in the simple past. The option C) i.e. “also got” seems
to fit perfectly. So the correct sentence can be written as “Besides her friends, her wife and colleagues also
got the invitation.
Note that “also got” is not the only correct option. We can easily put “also get” in there without making the
sentence grammatically incorrect.
Example 2: Great representatives are always empathetic towards her people’s needs.
A) A great representative is B) Great representatives is C) Some great representatives are
D) All great representatives are E) No correction required
Answer: The hint word or the defining word here is the possessive adjective ‘her’. This occurs in the
predicate part of the sentence (the part of a sentence or clause containing a verb and stating something
about the subject) containing this possessive adjective hints at a subject that is singular in nature. In other
words “representative is” will make the sentence grammatically more correct. Hence the option A) A great
representative is, is the correct option.

Solved Examples – Types II


Let us see some more solved examples of sentence improvements. The main aim is to check the grammar
of the sentence. Let us see some more examples and the rules that let us improve them.
Example 3: The office gives differently rate of interest depending on the quality of the work.
A) difference in the rate of interests B) difference what rate of interest C) different rates of
interest D) the different rate of interest E) No correction required
Answer: The sentence has a noun phrase in it namely “rate of interest”. Before a noun phrase, we can’t use
an adverb like differently because an adverb can only qualify a verb and not a noun. Thus the use of the
adverb here is wrong and we replace it with the adjective ‘different’. An adjective is what qualifies a noun.
So the correct answer is C) different rates of interest.
Example 4: It is too early to know how the effect will the new policies will have on the people.
A) what effect will B) that the effect will C) how much effects will
D) what are the effects of E) No correction required
Answer: In place of the adverb ‘how’, we must use the adjective ‘what’ to make the sentence grammatically
correct. We can write the correct sentence as “It is too early to know what effect will the new policies have
on the people.” It may also be written as “It is too early to know how the new policies will affect the
people.” With respect to the options that are present here, we see that the correct option is A) what effect
will.

Practice Questions
Q 1: Khan is enough kind to help me pack my stuff and move out.
A) Kinder enough
B) Kindly enough
C) enough kindly
D) kind enough
E) No improvement
Ans: D) Kind enough
Q 2: Some people think that mathematics is only a product of the human brain, and is not something that is
inherent in the Universe.
A) to the
B) for the
C) in the
D) with the
E) No improvement
Ans: A) to the.

Sentence Formation
 Sentence Formation is a subsection of the correct sentence section of IBPS PO, SO, SBI, RBI, SSC and
many other exams. In this section, we will see various types of sentences and methods to form such
sentences. We will also see how to detect and check for errors in the sentences with regards to the
sentence formation. In the section below, we will see the different types of sentences and the
various types of errors that can be present in them. Let us see!

 Sentence Formation
 A sentence is a meaningful collection of words. However, with the rules of grammar in mind, we can
say that a sentence is a group of words that has a subject, a verb, and an object. In English, this is
known as the SVO structure or the SUbject-Verb-Object structure. The subject comes first, then
comes the verb and, following which comes the object. Let us first see the different types of the
sentences. In the space below we will discuss definitions and examples of the various types of
sentences. Let us begin with what is known as the simple sentence.

 Source: Wikimedia

 Simple Sentence
 A sentence is a simple sentence if it contains only an independent sentence. This type of sentence
doesn’t contain any dependent clause. Let us see some of the examples:
 The dog barked. This is a simple sentence. Harsh takes a walk in the morning. The trains are late
today etc. are all simple sentences. All of these sentences have a single independent clause. Let us
see a sentence correction example of this section:
 Example 1: He always gets into trouble for hurt his classmates.
 A) hurting his B) having been hurt C) his hurt D) his hurting E) No correction
required.
 Answer: In the context of the sentence the correct option here would be A) hurting his. This is the
present continuous tense and is thus suitable with respect to the given sentence.
 Compound Sentence
 These sentences are of two clauses. Each of them is an independent clause, so we can say that
every combination of two independent clauses will form a compound sentence. The two clauses will
be connected with what we call the “FANBOYS” conjunctions. The FANBOYS conjunctions are the
following:

F for

A and
N nor

B but

O or

Y yet

S so

 All of these conjunctions signal that the sentence is a compound sentence. We call them the
coordinating conjunctions. For example, The teacher said good grades are for good students only.
He waited for her, but she had made her mind.
 Example 2: A man or a woman are not to be told what to do and what not to!
 A) is B) were C) might D) could E) No need for improvement
 Answer: The correct answer is A) is as the conjunction here is or.

 Complex Sentence
 The third type of sentences are the complex sentences. The complex sentences have one or more
than one dependent clauses. They may also contain an independent clause but must have at least
one dependent clause. The dependent clauses can be identified with the help of conjunctions that
are known as the subordinating conjunctions.
 Following are some of the examples of subordinating clauses: if, when, as, before, until, after, since,
while, though, before, even, though, unless etc. are all subordinating clauses that if put in front of a
clause making it a dependable clause. Examples of complex sentences are: If wishes were horses,
beggars would ride. It will not rain until August.
 Example 3: His admission was delayed if the summer internship doesn’t take him.
 A) is delayed B) was delay C) is delaying D) would be delayed E) No
improvement needed.
 Answer: The correct sentence here is: “His admission would be delayed if the summer internship
doesn’t take him.” So the correct option is A) is delayed.

 Other Sentence Types


 There are other sentence types that are based on the mood or the tempo of the speaker or the
writer. They are given below with examples:
 Assertive or Declarative Sentences: These are the sentences that state facts, or declare statements.
For example, India is a secular country. The sun rises in the East etc.
 Interrogative Sentences: The sentences that ask questions or inquire about something are
interrogative sentences. For example, Is she serious? Are you going to the party? etc.
 Imperative Sentence: These sentences convey orders, pieces of advice, orders, requests etc. For
example, Shut the door.
 Exclamatory Sentences: They express the strongest emotions of fear, excitement, happiness,
sorrow, shock etc. The exclamatory sentences often end in an exclamatory mark ‘!’. For example,
Alas! I forgot my phone.
 Practice Questions
 Q 1: Policies to improve the health sector, which is one of the worst in the world, requires
commitment from the country’s politicians.
 A) which is one of the worse in the world.
 B) which is among the worse in the world.
 C) who is one of the worst in the world.
 D) which is very bad in the world.
 E) No improvement needed.
 Ans: E) No improvement needed.
 Q 2: The Chief Minister lied when he said that he will give a job to all the unemployed.
 A) to each of the unemployed.
 B) to the unemployed.
 C) to many unemployed.
 D) to every unemployed.
 E) No improvement needed.
 Ans: A) to each of the unemployed.

Sentence Connectors

As per the IBPS exam pattern, Sentence Connectors section has a number of sentences. Some of these
sentences can be connected and form a more complex but grammatically correct sentence. Here we will
study tips and tricks to identify such sentence connectors. There are a number of options, each option is a
sentence or a clause. Your task is to identify these sentences or clauses and figure out which of these
sentences can be written together. Let us begin.

Sentence Connectors
Sentence connectors are phrases or words. These when put at the beginning of two sentences, connect
them in a way that retains their original meaning. This is what we call the coherence of the sentence
connectors. For example, consider the following:
Example 1: Directions: In the below question, two sentences are grammatically correct and meaningful.
Connect these by the words present in the options. Connect the statements in the best possible way such
that their coherence is not lost. Choose the answers accordingly from the options to form a correct and
coherent sentence(s).
Sentences – I: It is getting late.
II: We should leave.
i) Although. ii) Considering that. iii) Even though.
A) i) only. B) ii) only C) iii) only D) Both i) and iii) E) Both ii) and iii).
Answer: First of all note that the sentences have to be put together in the order they are present. That is to
say that we have to put sentence I first and sentence II second. Now if we write Although it is getting late,
we should leave, it will form a grammatically correct sentence. But it is not coherent. The sentence is
logically incorrect. We see that option iii) forms the following sentence: Even though it is getting ate, we
should leave. This seems to make sense. Therefore the correct option is E) Both ii) and iii) sentence
connectors that will form a coherent sentence.

The Mnemonic “ON A WHITE BUS”


We can use the mnemonic “on a white bus” to help us remember many sentence connectors that you will
encounter in the paper. Sentence connectors are also known as ” sentence synthesis” exercises. They were
first asked by SBI for their PO exams last year. We will state all the common sentence connectors here.

O Only if, Once…

N Now that, …

A After, Although, As, …

W Where, Wherever, When, Whenever, Whether, While, …

H How, …

I In case if, In order that, …


T Though, That,…

E Even Though, Even if, …

B Before, Because, …

U Until, Unless, …

S Since, So that,…

Sometimes you may have to change the order of the sentences to connect them. Consider the following
example.
Example 2:
In the below question, two sentences are grammatically correct and meaningful. Connect these by the
words present in the options. Connect the statements in the best possible way such that their coherence is
not lost. Choose the answers accordingly from the options to form a correct and coherent sentence(s).
Sentences – I: He goes home on a bus.
II: This is the bus.
i) He takes to ii) After. iii) Even though.
A) i) only. B) ii) only C) iii) only D) Both i) and iii) E) Both ii) and iii).
Answer: Consider the sentences first. The sentence I states a fact that we explain in the second sentence
itself. In the first option, the word to has been added. This is a hint word. We can connect the two sentences
as follows:
He goes home on a bus and this is that bus.
This is the bus he takes to go home.
We see that out of the three only option i) is a coherent sentence connector. Hence the correct answer is i)
He takes to.

Other Sentence Connector Questions


Sometimes a word or a phrase may be already present. In such questions, you will have to read and
connect a few sentences present in the options. Out of the many options, only a few will be such that they
can be coherently put together. You will have to choose these sentences only. Let us see an example of such
questions:
Example3: Despite:
i)the-reservations-of-the-school-principal.
ii)the-carbone-mission-decreasing-invarious-countries.
iii)the-globalrisein- temperatures has-notstopped.
iv)the-festival-passed-of-smoothly.
A)Onlyi)&iv).
B)Onlyii)&i).
C)Bothiii)&ii);iv)&i)
D)Onlyi)&iv).
Answer: Of the options that are present, we can form the following coherent sentences:
Despite the reservations of the school principal, the festival passed off smoothly.
Despite the carbon emission decreasing in various countries, the global rise in temperatures has not
stopped. As a result, we see that option C) is the correct option here.
Test yourself by practicing 20 Sentence correction questions here.

Practice Questions
Directions: In the below question, two sentences are grammatically correct and meaningful. Connect these
by the words present in the options. Connect the statements in the best possible way such that their
coherence is not lost. Choose the answers accordingly from the options to form a correct and coherent
sentence(s).
Q 1: I: He works from his garage.
II: He never leaves his house.
i) Considering that ii) Although iii) Even though iv) As such
A) i) only B) ii) only C) iii) only D) iv) only E) ii) and iii)
Ans: A) i) only.
Q 2: As long as:
i) the politicians are sincere
ii) the country will not be saved
iii) we are all going to be fine.
iv) we will never be able to sleep in peace.
A) Only i) and iii) B) Only i) and ii) C) Only i) and iv) D) Only ii) and iii)
Ans: A) Only i) and iii)

Phrase Replacement
In the Phrase Replacement sections, a candidate’s knowledge of the English language is put to test. In
this section usually, the candidate is asked to replace a phrase from a given sentence. This phrase can
be replaced by one of the phrases present in the options. This is a very important section and we will
try and do as many examples as we can. Let us begin.

 Phrase Replacement
 As we have already made clear, this is a very important section of the IBPS PO, SO exams. Let us first
begin by stating some important phrasal verbs here. These phrasal verbs are very important for any
candidate planning to appear for the IBPS PO, SO, SBI exams. Let us list them below:
 act up: behave or function improperly. Example: I think I need to take my car to the mechanic
because it’s acting up again.
 add up: calculate a sum. Example: I added up the receipts and got a total of 8000.
 add up to: Be equal an amount. Example: The total bill adds up to three thousand rupees.
 add up: make sense. Example: His story doesn’t add up. I think he is lying.
 ask out: invite on a date. Example: I can’t believe that Joe finally asked me out on a date!
 ask over: invite to one’s home. Example: Why don’t we ask the neighbours over for dinner?

 B
 blow up: make explode; destroy using explosives. Example: The terrorists blew the bridge up.
 blow up: suddenly become very angry. Example: When Harsh heard the news, he blew up and
rushed out of the room.
 break down: analyze in detail. Example: We need to break this problem down in order to solve it.
 break down: stop working properly. Example: The truck broke down in the desert.
 break down: become mentally ill. She broke down after her husband died.
 break in: wear or use something new until it is comfortable. Example: I need to break these shoes in
before I go hiking.
 break in: interrupt. Example: While we were discussing the situation, Terri broke in to give her
opinion.

 C
 call on: invite someone to speak at a meeting or a classroom. Example: Professor Jim called on Tim
to answer the question.
 care for: like someone or something. Example: I don’t care for sour cream on my potato.
 carry on: continue (a conversation, a game). Example: Please, carry on. I didn’t mean to interrupt
you.
 catch on: slowly start to understand. Example: He caught on after a few minutes.
 catch up: make up for lost time. Example: I will never catch up. I am too behind in my work.
 catch up with: speed up to be at the same place as a person or thing in front of you. Example: I had to
run to catch up with the others.
 catch up on: become up-to-date. Example: I need to catch up on world events.
 check by: go to a place to see if everything is OK. Example: We need to check by the office to see if the
documents are ready.

 D
 die down: diminish in intensity. Example: The controversy about the president’s affair finally died
down.
 die off/out: become extinct. Example: Whales are in danger of dying off.
 disagree with: cause to feel sick due to food or drink. Example: Spicy food disagrees with me.
 do away with: abolish. Example: Most rational people want to do away with the death penalty.
 do over: repeat. Example: You made many mistakes, so I want you to do the report over.
 do without: manage without something one wants or needs. Example: I couldn’t do without a car in
California.
 draw up: create ( a contract). Example: Let’s draw an agreement up before we go any further with
this project.
 dress down: reprimand severely. The mother dressed her son down for skipping school.
 dress down: dress casually. Example: I am dressing down because we’re going to a barbecue by the
beach.
 dress up: decorate. Example: You could dress this house up with some bright colors.
 dress up: wear elegant clothes. She always dresses up at work.
 Now let us see some examples.
 Expected Questions
 Directions: In the following questions, select a suitable replacement for the word in bold.
 Q 1: I am used to hard work.
A) work hard B) working hard C) working D) no improvement
 Answer: The correct sentence will be ” I am used to working hard”. Work hard is a phrasal verb
while as hard work is an adjective clause. Hence the correct option is A) working hard.
 Q 2: I cannot listen what she is saying.
A) hear what B) listen for what C) listen to that D) no improvement
 Answer: The correct sentence is “I can’t hear what she is saying.” So the correct option here is A)
hear what.
 Practice Questions
 Q 1: A country will cease to be a member of the EU two years after the trigger of Article 50 unless it
reaches an agreement with the European Council to extend that period.
A) triggering of B) triggered of C) triggering on D) triggering in E) No improvement
is required
 Ans: A) triggering of
 Q 2: Nostalgia gripped the veterans as they spoke about small incidents, painful moments, happy
times, togetherness and bonding during the sessions that was engaged and engrossing.
A) was engaging B) were engage C) were engaging D) are engaging E) No
improvement is required
 Ans: C) were engaging.

 Common Errors in Spoken English


Sentence Correction Practice Questions
Sentence Correction Practice Questions section has all the important types of questions that you require to
ace this section. A sentence is the basic unit of language. A sentence is a group of words that have a syntax
and convey a certain meaning. A complete knowledge of a sentence and its structure is necessary to spot
any error in the sentence and vice versa. As questions on sentence correction are very frequent in several
competitive exams, we will cover such questions here in this section. There are many types of errors that
can occur in a sentence. The Correct Sentences Practice Questions will try to cover most of them, giving you
a headstart against the competition.

Sentence Correction Practice Questions

Type I
Directions: In each of the following questions, find out which part of the sentence has an error. If there is
no mistake, the answer is “no error”.
Q1: The road (a) / to famous monument (b) / passes through a forest (c) / No error (d).
Q2: Our housing society comprises of (a) / six blocks and (b) / thirty-three flats in an (c) / area of about
thousand square metres. (d) / No error (e).
Q3: Now that she is living in her own flat, (a) / she cleans the windows, (b) / twice a week in the summer
and once a week in the winter. (c)/ No error (d).
Q4: With little imagination and enterprise, (a)/ the tournament could have been transformed (b) / into a
major attraction (c) / No error (d).
Q5: These display (a) / the (b) / remarkable variety. (c) / No error (d). [N.D.A.]
Q6: If you have faith in Almighty (a) / everything will turn out (b) / to be all right. (c) / No error (d).
Q7: According to the Bible (a)/ it is meek and humble (b) / who shall inherit the earth. (c) / No error (d).
Q8: I am not wealthy, (a)/ so I can not afford (b) / to buy a expensive car. (c) / No error (d).
Q9: Sunita opened a almirah (a) / full of books (b)/ and took one of them (c) / for reading. (d) / No error
(e). (B.S.R.B 1992)
Q10: Most people would have (a) / attended the union meeting (b) / if they had (c) / had longer notice of it.
(d) / No error (e)
Q11: My father is (a) / in bad mood (b0 / today. (c) No error (d).
Source: Wikimedia

Find Your Answers Here


Q1: (b), Q2: (d), Q3: (c), Q4: 9a), Q5: (b), Q6: (a), Q7: (b), Q8: (c), Q9: (a), Q10: (d), Q11: (b).

Type II
Directions: The following sentences may contain errors in grammar, usage, diction (choice of words)
or idiom. No sentence contains more than one error. You will find that the error if there is none, is
italicised and numbered. Assume that all other elements of the sentences are correct and cannot be
changed. If there is an error, select the italicised part that must be changed to make the sentence correct.
The number of that italicised part is your answer. If there is no mistake, the answer is “No error”.
Q1: The brand proposition now therefore had to be (1) / that Keo Karpin Antiseptic Cream is more
effective (2) / because it penetrates deep down (being light and non-sticky) and works from within (3) /
(because of its ayurvedic ingredients) to keep skin blemish, free and helps cope with cuts, nicks, burns and
nappy rash. No error (4). [MBA]
Q2: It is argued that if the income of capitalists is taxed sufficiently (1) / by putting a ceiling to (2) / the
maximum income of capitalists and by introducing legislation for the welfare of labourers and by
freezing prices, the aims of socialism will be realised (3) / No error (4).
Q3: The average student, devoid of (1) / good accommodation or atmosphere for study at home, utilises
public libraries for (2)/ a great advantage to himself (3) / No error (4).
Q4: Whereas Gandhi appealed to (1) / the good sense of the human nature and sought after bringing out
(2) / the kinder elements in the capitalists and other selfish individuals, Nehru believed in a legally
regulated way of disciplining (3) / the selfish elements in society. No error (4).

Find Your Answers Here


Q1: (1), Q2: (2), Q3: (2), Q4: (2).

Type III
Directions: In each sentence below, four words/ groups of words which are labelled (a), (b), (c), and (d)
have been printed in bold type. One of which may be either inappropriate in the context of the sentence or
wrongly spelt. If all the four are appropriate and also correctly spelt, mark ‘All correct’ as the answer.
Q1: The exhistence (a) / and attributes (b) / of the colonial (c) / army were decisive (d) / to the outbreak
of Civil War. All correct (e).
Q2: The success of a man (a) / in business (b) / depends over (c) / his attention to small things (d) / All
correct (e).
Q3: No country (a) / can isolate (b) / itself from international (c) / politics (d) / All correct (e).
Q4: He appears (a) / to be unreasonably (b) / anxious (c) / about solving his neighbour’s (d) / problem. All
correct (e).
Q5: A very important secret of (a) / the art of writing (b) / biographies (c) / is that the secondary
characters must be deleneated (d) / with the same care as the central figure. All correct. (e)
Q6: The peculiar (a) / dress he wears (b) / gave him a comical (c) / appearance (d) / All correct (e).
Q7: If a country is to prosper, (a) / it must produce goods both for domestic (b) / consumption (c) / and
for imports abroad (d) / All correct (e).

Find Your Answers Here


Q1: (a), Q2: (c), Q3: 9a), Q4: (d), Q5: (d), Q6: (b), Q7: (d).

Vocabulary:-

Antonyms and Synonyms


Antonyms and Synonyms are asked very frequently in the IBPS exam. They may appear as part of some
other section or maybe in ways that are not straightforward like we will see here. An antonym is a
word that has the opposite meaning as that of a given word. On the other hand, a synonym is a word
that has a completely similar meaning as that of a given word. Here we will state some examples and
try and develop some vocabulary.

 Synonyms
 The Synonyms and antonyms are an important concept to learn while you are working on your
vocabulary. Synonyms are often used to flavour the text and make it diverse and colourful.
Synonyms and Antonyms are featured in many competitive exams and thus we have this section
dedicated to the concepts of Synonyms and Antonyms.
 Your objective here is to be able to develop a strong vocabulary. We will state some words and their
synonyms that may appear in a graduate level examination. However, to qualify this section, you
will have to develop a very strong vocabulary by memorizing at least a few words and their
synonyms and antonyms every day till the day of your exam. Let us see some examples of synonym
words here.

A B

Quick-
Abandon: Leave Forsake Bright: Vivid
witted

Abridge: Shorten Curtail Baffle: Deceive Confuse

Abundant: Plentiful Copious Brazen: Blatant Barefaced

Acceptable: Satisfactory Appropriate Bizarre: Aberrant Ludicrous

C D

Callous: Hard-bitten Case-hardened Diabolic: Fiendish Demonic


Congenial: Comradely Companionable Describe: Portray Recount

Confluence: Conflux convergence Definite: Obvious Distinct

E F

Ersatz: Adulterated Spurious Friendly: Comradely Companionable

Espionage: Reconnaissance Tailing Foreboding: Apprehension Consternation

Eccentric: Aberrant Peculiar Fast: Lickety-split Expeditiously

G H

Guilt: Culpability Delinquency Hypocrisy: Falseness Duplicity

Genial: Affable Amiable Horrible: Harrowing Gruesome

Gloomy: Sad Wretched Hate: Abominate Despise

I J

Illiterate: Ignorant Unlearned Jovial: Jolly Merry

Imperious: Authoritative Dictatorial Judicious: Discreet Prudent

K L

Knavery: Prank Roguery Lethargy: Sluggishness Torpor

 We encourage you to build on similar vocabulary through memorizing exercises. Now let us see
some examples.
 Synonyms
 Example 1: The food that was served was rather unpleasant, but her company made up for
everything.
 In the above sentence, the word(s) in bold can be replaced by which of these words.
 A) Vexatious B) Torpor C) Jolly D) Aberrant
 Answer: A lot of books and online articles that you read while preparing for bank exams will tell you
that there are rules and shortcuts that will allow you to guess the synonyms or antonyms of the
words that you come across in a sentence. However, this is not true. The only strategy that works
with vocabulary is to start learning new words. A few words each day should do the trick. As far as
this example goes, you will see that the words do sound similar.
 We are looking for a word whose meaning is somewhat similar to unpleasant. Vexatious means
something that is very frustrating or something that annoys. We reject this an consider this as a
viable option. Jolly means something that is pleasant and jocular, so jolly is rejected. Let us first see
the other two. Torpor, although sounds bad but it actually means lethargy or laziness. So this is also
not the correct option. Finally, aberrant means something that is diverging, it may be unpleasant
but that is not what the word means. Hence the correct option is Vexatious.
 Antonyms


 Let us consider the following questions:
 Q 1: “In a politically enlightened community, neither the Kings nor the gods have anything to say!”
 The word in the bold is exactly opposite in meaning to?
 A) Civilised B) Stupid C) Uncivilised D) Uneducated
 Answer: At first you may think that you want to find the words that have the opposite meaning to
enlighten. But wait, three of the options have a meaning exactly opposite to enlighten. So which one
to choose. The context matters here and if you have been asked to pick a word from a given
paragraph, you will have to guess its meaning in the context.
 For example, in the given sentence, the word enlightened has been used as informed or educated, so
the opposite word from the options would be uneducated. The answer thus is Uneducated.
 Remember the more you work on your vocabulary, the better you will get. Also, when you are asked
to find the synonym or the antonym of a word, take the context into consideration. If you don’t know
the meaning of the word, you may be able to guess it from the context sometimes.
 Practice Questions
 Q 1: Read the following paragraph and answer the question that follows,
 Most of the human history has been the Paleolithic period or the old stone age. There is also a
mesolithic period that comes about 20000 years ago. The exponential change in the development of
the human civilization, however, came in the recent few centuries.
 The word in the bold can be replaced by the following word without changing the meaning of the
sentence.
 A) Tremendous B) Fast C) Great D) Increasing
 Ans: A) Tremendous.
 Q 2: Read the following paragraph and answer the following question
 “These bottles are harrowing. How could you spend so much money on something that is so drab?
You should have called me and confirmed at least. This is what you always do!”
 Which of the following words represents the antonym for the word in bold?
 A) Gruesome B) Horrible C) Attractive D) Adorable
 Ans: C) Attractive.

Odd one out

The odd one out section is a very scoring section of the vocabulary section. This section is asked majorly in
the IBPS PO, IBPS SO, SBI, RBI, and other similar exams. The questions in the Odd one out section are
designed to check your vocabulary. The questions are generally formulated in a way where you are
provided with a set of given words that are related in some way. There is one option that doesn’t relate to
the other words in the same way, which becomes the Odd one out.

Odd one out


Let us see what does odd one out means with the help of some examples.

Type I Questions
These questions are based on the straightforward word-meaning logic. The words are provided in groups
and you are asked to pick out the odd word or the odd group. Let us see some examples.
Q: Pick the option that doesn’t belong to the following classification.
A) Caustic B) Brazen C) Vitriolic D) Mordant
Answer: This will tell you that there are no shortcuts to a good vocabulary. Let us see the options one by
one. Caustic means something that is corrosive, a mordant. Vitriolic means something that is bitter or
unpleasant. Hence the three words have a somewhat similar meaning. Now we have brazen which means a
thing or a person which is bold and without shame. Hence the odd one out here would be B) Brazen.

Type II – Sentence Based


These type of questions will give you a set of sentences and will ask you to pick a sentence that doesn’t
belong to the given set of questions. Let us see some examples of the same.
Q: The following sentences are labelled A, B, C, and D. They form a paragraph and/or a logical set. Pick out
the one that doesn’t belong.

A. In Mesopotamia, we had the Syrian Kingdom, thriving during the Shang dynasty.
B. The Shang dynasty was a copper age civilization.
C. The bronze civilization is a recent age in the hierarchy of humans.
D. The weapons used by the Shang dynasty were made of copper and were very advanced.
Answer: If you read the sentences aloud, they are all talking about either the Shang dynasty or the copper
age civilizations except the option C which talks about the Bronze civilizations. Hence the odd sentence
here is C.
Q: The following sentences are labelled as A, B, C, and D. They form a paragraph and/or a logical set. Pick
out the one that doesn’t belong.

A. India is often described as the world’s largest democracy.


B. A democracy is a system where the government is supposedly run by the popular will.
C. Although the democratic system of governance is not perfect, it is the best system that we can have
right now.
D. All the other systems of governance limit the rights and freedom of an individual in one way or the
other.
Answer: The options from A through D talk about a democracy. But the option A talks about the democracy
in India, a concept that is not shared by any of the other options provided in the question. hence the
correct option is A. India is often described as the world’s largest democracy.

Type III – Mixed Questions


In these questions, you will find sentences with rather convoluted words and meanings. You will have to be
very careful even though the possibility of such question being asked is very low, but it is better to be
ready. Let us see an example.
Q: The following sentences are labelled as A, B, C, and D. They form a paragraph and/or a logical set. Pick
out the one that doesn’t belong.

A. We haven’t been able to decipher the writing of the Indus valley civilizations which existed right
before the Vedic period.
B. In the Vedas, which followed the Indus valley civilization, we find a direct reference to a stratified
society.
C. The classification that segregated the social classes on the basis of their birth, finds its roots in the
Rigveda.
D. However, there is an opinion that the caste system is a post-Vedic phenomenon.
Answer: All of the sentences talk about the Vedas. So which one is the one that doesn’t belong? Consider
each sentence one by one.
A. This option talks about the Indus valley civilization and its relation to the Vedic period. The main
subject is, however, the Indus valley civilization.
Option B through D, talk about the Vedas and the caste system. In all of these sentences, the caste system
or the stratification of the society is the main topic, so these are related to each other. hence, the correct
option here is A.

Practice Questions
Q 1: Pick the option that doesn’t belong to the following classification.
A) Salmagundi B) Gallimaufry C) Farrago D) Portfolio
Ans: D.
Q 2: The following sentences are labelled as A, B, C, and D. They form a paragraph and/or a logical set. Pick
out the one that doesn’t belong.
A. We believe that many modern languages of Europe and Asia are a subset of a larger mixture.
B. Some linguists argue that this is as a result of the fact that most of the civilizations originated from a
smaller group of people who had their own language.
C. Similar to it is the fact that when the Europeans colonized America, they bought their cultural
influence with them.
D. Sanskrit words can be found in many languages including Greek, English and many Indian
languages.
Ans: C.

Idioms
Idioms/Phrases are widely asked in the bank PO, SO, SBI and RBI exams. In this article, we will list
many phrases, use them in sentences and see what kind of questions are based on the knowledge of
these Idioms/Phrases. All of the phrases that we will use in the articles have come at one point or the other
in one of the exams that we are preparing for.

Idioms
The dictionary definition of the word idiom is that idiom is a group of words established by usage as
having a meaning not deducible from those of the individual words. It is an expression that has a specific
meaning. For example, a sweeping statement is an idiom. It sounds like a statement that is very
impressive, however, its meaning is exactly opposite. To make a sweeping statement means to make a
thoughtless statement. So don’t be fooled by the people who tell you that there is a rule or a method that
tells you what a given idiom means.
As this section forms the part of the vocabulary, you will have to memorize these idioms and their
meaning. Let us state some idioms that have already been asked in the exams that we are preparing for.

Idioms and Their Examples

Let us state some examples that you will commit to your memory.

 All at sea: Puzzled.


 A White Elephant: Costly or troublesome possession.
 A Wild Goose Chase: Futile search.
 At One’s Wit’s End: Puzzled / Confused /Perplexed.
 A White Elephant: A costly but useless possession.
 An Axe To Grind: A private interest to serve.
 An Open Book: One that holds no secrets.
 A Bolt From The Blue: Something unexpected and unpleasant.
 A Sore Point: Something which hurts.
 A Red Letter Day: An important day.
 At Stone’s Throw: Very near.
 A Dark Horse: An unforeseen competitor.
 A Man In The Street: An ordinary person / Common man.
 By Fits And Start: Irregularly.
 By Leaps And Bounds: Very Quickly.
 Beat About The Bush: Circumlocution / Does not talk specifically.
 Reading Between The Lines: Looking for meanings that are not explicitly present in a context.
Similarly, we shall try and put forward a few such examples here. These examples will help us to know
many idioms and their usage.

Solved Examples – Type I


Example 1
To take with a grain of salt.
A) Good and evil
B) Former and latter
C) To take with some reservation
D) To destroy before it blooms
Answer: To take something with a grain of salt is to have reservations about something that you are
expected to be a party to. Thus the correct answer here will be C) To take something with some
reservation.
Example 2
Let something slip through one’s fingers.
A) To waste an opportunity to achieve something.
B) Start the day in a bad mood, which continues all day long.
C) To uselessly dwell on a subject far beyond its point of resolution.
D) In anxious suspense.
Answer: The meaning of letting something slip through one’s fingers is to waste an opportunity to achieve
something. So the option that is correct here is A)

Solved Examples – Type II


In the following examples, select the correct representation of the word or the phrase.
Example 3
He took the decision of his own accord.
A) Willingly B) At his desire C) Unwillingly D) On his own.
Solution: The meaning of doing something on your own accord is to do something that one does with his
own desire.
Example 4
The people who think that there can be any prosperity without freedom are still in a fool’s paradise.
A) Living in illusions B) In a good place C) Being among idiots D) Living with nice
ideas.
Answer: To live or to be in a fool’s paradise means to be living in an illusion. So the answer is A).
Example 5
The order disqualifying the promotion of the clerk is kept in abeyance.
A) In suspension. B) To strengthen C) Bring harmony D) To revolt.
Answer: To be in abeyance means to be in a state of suspension. So the correct option here is A) In
suspension.
Example 6
I think the politician’s account of his education is a cock and bull story.
A) Absurd and unbelievable story.
B) Very strong and opinionated.
C) Very irrational and evil.
D) Uninteresting and unimaginative
Answer: A cock and bull story is a story or an account by a person which is absurd and unbelievable. So the
answer is A) Absurd and unbelievable story.

Practice Questions
Q 1: What do you understand by the following sentence: “The President seems to have a seamy side”.
A) The President is a hardworking guy.
B) The President is an idiot.
C) The President is an unpleasant and immoral person.
D) The President has a tattoo.
Ans: C) The President is an unpleasant and immoral person.
Q 2: Pick the option that represents the meaning of the following sentence: The Economic Policies and the
Social Policies should never be at loggerheads.
A) The Economic Policies and the Social Policies should be similar.
B) Economic Policies and the Social Policies should not be similar.
C) These Policies should be in disagreement with each other.
D) The Economic Policies and the Social Policies should never be in disagreement with each other.
Ans: D) The Economic Policies and the Social Policies should never be in disagreement with each other.
Phrases
The dictionary meaning of a phrase is “a small group of words standing together as a conceptual unit,
typically forming a component of a clause.” Phrases are a group of words that have a known meaning. You
may say that Phrases and idioms are the same, but there is a difference. We will learn the difference and
use many phrases in the article given below.

Suggested Videos

Difference Between Phrase and Idiom


Both the Phrases and the Idioms are a collection of words that have a known meaning. The difference is
that in an idiom, the meaning arises from common usage. While a phrase is a small group of words that
have a meaning when taken together, out of the sentence. In order to check whether a given group of
words is a phrase or an idiom, a simple test can be done.
Take the set of words that you have been asked to check and see if they make any sense. If the meaning of
this group of words is exactly what you infer from the group, then it is a phrase. However, if the meaning is
different then it is an idiom. Let us see this with the help of an example.
I: It is raining cats and dogs.
II: I saw a herd of cats and dogs while crossing the road.
Select the correct option that represents the words in the bold alphabet:
A) Both I and II are idioms.
B) I is a phrase and II is an idiom.
C) II is a phrase and I is an idiom.
D) They are clauses.
Answer: The answer is obviously C) “Cats and dogs”, is an idiom while as a herd of cats or a heard of dogs is
a phrase, but there is a catch here. “Cats and dogs”, is a group of words that has a given meaning! In other
words, we say that every idiom can be treated as a phrase but every phrase is not an idiom.

Some Common Phrases

Let us see a few examples of questions that may appear in your paper, based on the concepts of phrases.
But first, let us see some commonly used phrases of the banking exams.

 A Picture paints a thousand words: Means that a visual representation of an event or happening is
always very descriptive.
 A wolf in sheep’s clothing: Someone whose appearance is a deception. A person who is dangerous
but pretending to be friendly.
 Above Board: Something fair and honest.
 Against the Clock: Rushed and short on time.
 Actions speak louder than words: It is better to act than to talk about action.
 Back to the drawing board: When an attempt fails and it’s time to start all over.
 To bite off more than you can chew: To take on a task that is way too big.
 Can’t judge a book by its cover: Cannot judge something primarily on appearance.
 Don’t count your chicken before the eggs have hatched: Don’t make plans for something that is not
possible.
 Get over it: To move beyond something that is bothering you.
 Get up on the wrong side of the bed: Someone who is having a horrible day.
 Being a good Samaritan: Someone who helps others when they are in need, with no conditions.
 Hit below the belt: Contrary behaviour to the principles of fairness.
 Hit the nail on the head: Do something exactly right or say something exactly right.
 The icing on the cake: Something is an icing on the cake when it improves on something that you
already have or in other words if you already have it good and get something on top of what you
already have, it is an icing in the cake.

Solved Examples
Example 1: Read the following sentences and answer the questions that follow at the end.
I: Einsteins’ theory of relativity was a challenge of imagination.
II: We are to meet each other face-to-face.
A) I is a phrase and II is an idiom.
B) II is a phrase while II is the idiom.
C) They are both phrases.
D) They are both idioms.
Answer: The answer is A) challenge of imagination is a phrase whereas face-to-face is an idiom.
Example 2: What is the meaning of the phrase in bold?
The scientist explained his thesis in the worst way possible. It was all Latin and Greek.
A) Very strange B) It was difficult C) The thesis was impressive D) The thesis was in Greek.
Answer: The phrase “Latin and Greek” is used for something that is very difficult to understand. So the
answer here is B) It was difficult.

Practice Questions
Q 1: Someone is adding “fuel to fire” if they are:
A) Putting out the fire.
B) Starting a fire.
C) Want a situation, that is bad to get worse.
D) Want to help settle a situation that is very hot.
Ans: C) Want a situation, that is bad, to get worse.
Q 2: Deep down, he knew that he was wrong.
The words in the bold can be best described by the option:
A) In the depth of a well.
B) The person lives in a deep place.
C) This means the deep secrets of a person.
D) It means the deep feelings of a person.
Ans: D) It means the deep feelings of a person.

One word substitution


One word substitution means to find a single word that describes the given definition or property. In one-
word substitution questions, a definition will be given and you will be asked to choose a word for this
definition from the options that are provided. Let us see with the help of some examples.

One Word Substitutions


Let us see one-word substitution for A, B, and then C.
Let us state a few words and their definitions in alphabetical order i.e. to say that we will begin with A and
go up to C. Note that all of these words have been asked in the exams before. They have been asked in
the banking exams and other similar exams in the past.

Source: Erewise

One Word Substitution for A and B are as follows

 Amoral: One not concerned with right or wrong.


 Ambiguous: Capable of being understood in either of two or more possible senses and therefore not
definite.
 Anarchist: One who believes in no government and therefore incites disorder in a state.
 Autopsy: Dissection of a dead body to find the cause of the death.
 Archive: A place where government / public records are kept.
 Apathy: Lack of feeling.
 Arbitrator: Someone who is designated to hear both sides of a dispute and make a judgement.
 Burglar: A person who breaks into houses in order to steal.
 Boulevard: A broad road bordered with trees.
 Bohemian: One who does not follow the usual way of life.
 Bibliophile: One who loves books.
 Ballad: Poem in short stanza narrating a popular story.
 Bevvy: A group of girls.
 Brittle: Liable to be easily broken.
 Bovine: Affecting or relating to cows.

One Word Substitution for C is as follows.

 Conceited: To have a very high opinion of oneself.


 Cemetery: Place of burial (especially not in the churchyard).
 Caricature: A picture of a person or a thing drawn in such a highly exaggerated manner to cause a
laughter.
 Cosmopolitan: One who is a citizen not of a country but of the world.
 Chronological: Arrangement in order of occurrence.
 Conscience: Feeling inside you which tells you what is right and what is wrong.
 Calligraphy: The act of producing beautiful handwriting using a brush or a special pen.
 Claustrophobia: Fear of closed space.
 Congregation: An assembly of worshippers.
 Cellar: An Underground place for storing wine or other provisions.
 Congregation: People at a religious gathering.

Questions For the Alphabets (D – Z)


Now that we have stated a few words, let us do some questions.
Q 1: “To look at someone in an angry or threatening way” can be represented by?
A) Fatal B) Prevaricate C) Epitaph D) Glower
Answer: Let us see the meaning of each word.

 Fatal: Something that causes death.


 Prevaricate: This means to Quibble (to avoid giving a direct answer to a question in order to hide the
truth).
 Epitaph: An inscription on a tomb.
 Glower: To look at someone in an angry or threatening way.
Thus the correct option here is Glower.
Q 2: What is the one-word substitution to the following: “To feel or express disapproval of something or
someone”?
A) Deprecate B) Commemorate C) Amphibians D) Bilingual
Answer: Let us see the meaning or definition of each word.

 Deprecate: To feel or express disapproval of something or someone.


 Commemorate: To keep a great person or event in people’s memory.
 Amphibians: Animal that can live on land and in water.
 Bilingual: A person who is fluent in two languages.
So here the answer is Deprecate.

Write the one-word substitution for the following questions.


Q 3: Can you substitute a word for: “A man who wastes his money on luxury”.
A) Notorious B) Versatile C) Ornithologist D) Extravagant.
Answer: Let us see the meaning of the words one by one in the below space.

 Notorious: A person who is well known in an unfavourable way.


 Versatile: Someone having many skills.
 Ornithologist: Someone who scientifically studies the birds.
 Extravagant: A man who wastes his money on luxury.
Hence the correct option here is Extravagant.
Q 4: Choose the correct word that represents, “A political leader appealing to popular desires and
prejudices.”
A) Dictator B) Truant C) Triumvirate D) Demagogue
Answer: Let us see the meaning of each option and then declare one of these options that is correct.

 Dictator: A ruler with total power over a country, typically one who has obtained control by force.
 Truant: A schoolboy who frequently cuts classes.
 Triumvirate: A group of three powerful people.
 Demagogue: A political leader appealing to popular desires and prejudices.
The correct option is Demagogue.

Practice Questions
Q 1: Write the one-word substitution for: “Murder of a brother”.
A) Fratricide B) Effeminate C) Truism D) Altruist.
Ans: A) Fratricide.
Q 2: What word from the following given words would you use to represent “A doctor who specializes in
diseases of the nose”.
A) Rhinologist B) Extempore C) Speleology D) Chauffeur.
Ans: A) Rhinologist.

Fill in the blanks


 Fill in the blanks section of the exam will prompt the candidates to use their vocabulary to complete
sentences that have missing words in them. The words that go in the blanks could be anything. They
could be models, articles, words, phrases, idioms etc. Here we will see all types of questions that you
may encounter in the exam. Let us begin!

 Fill in the Blanks

 Type I
 In this type of questions, the sentences will be given and you will be prompted to find the options
that best fit the blanks. Complete the following sentences using the words given at the end.
 1. The most celebrated tin mines ____ Cornwall are, still, close ____ the sea.
 A) in, to B) at, to C) in, in D) in, at
 Answer: To select the correct option, always make sure that after you have selected a suspected
option, you read out the sentence aloud and try to see which option makes the most sense. Cornwall
is a place and thus the correct article to go in would be “in”. Also with the sea, we use to which shows
that the mines are closed. Therefore the correct option is A) in, to and the complete sentence will
be: “The most celebrated tin mines in Cornwall are, still, close to the sea.”

 2.Complete the following sentence using the words given at the end
 The whole country was ______ with forests and swamps. The ______ part of it was very misty and cold.
 A) made, Pointed
 B) covered, Greater
 C) Playing, Likely
 D) Filled, Snake
 Answer: Again use the options that seem appropriate and read out the sentence once. The option
for the first blank should be “covered” as the other words are less suitable. Similarly, the option that
goes in the second blank will be “greater” as can be seen from the option that is B) covered, greater.

 Type II
 There may be excerpts from novels and books. These paragraphs would have a meaning and the
missing word will be crucial to the completion of the meaning in one way or the other. Let us
consider the following solved examples.
 Example 1: At eleven he had a horror of death. Within six impressionable years, his parents had
died and his grandmother had faded off almost _______, until, for the first time since her marriage, her
person held for one day an unquestioned supremacy over her own drawing room.
 A) imperceptibly B) suddenly C) Tragically D) violently
 Answer: When you answer questions like these make sure that you read all the parts of the
paragraph. For example, here if you focus only on the first part only, you will see that every option
fits the blank perfectly. So for the answer, the first part is not relevant. In the second part, you
should see that the sentence talks about the grandmother. It says that one day, for the first time the
grandmother ” held an unquestioned supremacy.” This hints at the fact that the death of the
grandmother was not a crucial or a great impact on the subject of the sentence. So the blank should
be filled with something of a similar meaning. Something not that important or in other words A)
imperceptible.

 Type III
 There will be questions that require you to read a complete paragraph to be able to answer
correctly. However, there will always be a hint, either in the beginning or at the end of the
paragraph that should open it up for you.
 Read the following paragraph and fill in the blanks.
 At sixteen he had lived almost entirely within himself, a _______ boy, thoroughly un-American, and
politely bewildered by his contemporaries. The two preceding years had been spent in Europe with
a private tutor, who persuaded him that Harvard was the thing; it would “open doors,” it would be a
tremendous tonic, it would give him innumerable self-sacrificing and devoted friends. So he went to
Harvard–there was no other logical thing to be done with him.
 A) unfriendly B) unimaginative C) inarticulate D) unpleasant
 Answer: If you read the paragraph, and try out each option once, you will see that the option is C)
inarticulate. The reason for it is in the first sentence itself. The first part says that the boy lived
almost entirely within himself. That means he was shy. The rest of the paragraph qualifies this
quality. So the correct option is C).
 Practice Questions
 Q 1: Fill in the blanks in the following sentence: “He learned that he was looked upon as a rather
romantic figure, a scholar, a recluse, a tower of ______.”
 A) erudition B) height C) Ego D) Beauty
 Ans: A
 Q 2: Read the following paragraph and fill in the blanks.
 “He returned to America in 1912 because of one of his grandfather’s sudden illnesses, and after an
excessively tiresome talk with the perpetually _____ old man he decided to put off until his
grandfather’s death the idea of living permanently abroad. After a prolonged search, he took an
apartment on Fifty-second Street and to all appearances settled down.”
 A) convalescent B) convergant C) talkative D) repulsive
 Ans: A

Multiple Meaning Words or Homonyms

Multiple Meaning Words: This is going to be a really interesting chapter! Homonyms, or multiple–meaning
words, are words that have the same spelling and usually sound alike but have different meanings (e.g.
Bark– dog bark, tree bark). Right from kindergarten, we learn to use context to determine
which meaning of a multiple–meaning word is correct in a sentence.
Let’s check out a few examples of this interesting English language wonder, Multiple Meaning Words or
Homonyms:

 I left my phone on the left side of the room.


 The baseball pitcher asked for a pitcher of water.
 The committee chair sat in the center chair.
 While they are at the play, I’m going to play with the dog.
 She will park the car so we can walk in the park.
 The crane flew above the construction crane.

Understanding Multiple Meaning Words or Homonyms


In linguistics, homonyms, broadly defined, are words which sound alike or are spelled alike but have
different meanings.
A more restrictive definition sees homonyms as words that are simultaneously homographs (words that
share the same spelling, regardless of their pronunciation) and homophones (words that share the same
pronunciation, regardless of their spelling)– that is to say, they have identical pronunciation and spelling,
whilst maintaining different meanings.
Learn more about Synonyms and Antonyms here in detail.
The relationship between a set of homonyms is called homonymy. Examples of homonyms are the pair
stalk (part of a plant) and stalk (follow/harass a person) and the pair left (past tense of leave) and left
(opposite of right).
A distinction is sometimes made between true homonyms, which are unrelated in origins, such as skate
(glide on ice) and skate (the fish), and polysemous homonyms, or polysemes, which have a shared origin,
such as mouth (of a river) and mouth (of an animal).
In non-technical contexts, the term “homonym” may be used (somewhat confusingly) to refer to words
that are either homographs or homophones.
The words row (propel with oars) and row (argument) and row (a linear arrangement of seating) are
considered homographs, while the words read (peruse) and reed (waterside plant) would be considered
homophones; under this looser definition, both groups of words represent groups of homonyms.
The adjective homonymous can additionally be used wherever two items share the same name,
independent of how close they are or aren’t related in terms of their meaning or etymology.
A few more examples of Multiple Meaning Words or Homonyms: forearm, bat, beam, cast, command, duck,
dust, employ, even, flat.

Other such Multiple Meaning Words

1. Homographs (literally meaning “same writing”) are usually defined as words that share the same
spelling, regardless of how they are pronounced. If they are pronounced the same then they are also
homophones (and homonyms) – for example, bark (the sound of a dog) and bark (the skin of a tree).
If they are pronounced differently then they are also heteronyms – for example, bow (the front of a
ship) and bow (a ranged weapon).
2. Homophones (literally “same sound”) are usually defined as words that share the same
pronunciation, regardless of how they are spelled. If they are spelled the same then they are also
homographs (and homonyms); if they are spelled differently then they are also heterographs
(literally “different writing”). Homographic examples include rose (flower) and rose (past tense of
rising). Heterographic examples include to, too, two, and there, their, they’re. Due to their similar yet
non-identical pronunciation in American English, ladder and latter do not qualify as homophones,
but rather synophones.
A further example of a homonym, which is both a homophone and a homograph, is fluke. Fluke can mean:

 A fish, and a flatworm.


 The end parts of an anchor.
 The fins on a whale’s tail.
 A stroke of luck.
These meanings represent at least three etymologically separate lexemes but share the one form, fluke.
How about you go ahead and find such cool words today. It will be fun framing them in different sentences.
Have fun with grammar.

Practice Questions for You


1] Pick the correct homonyms for the given sentences.
Q: The king was in desperate need of a ______ to secure the future of his kingdom. (air or heir)
Ans: Heir. The king was looking for a successor.
Q: She ____ the ball so far that it landed in the neighbour’s garden. (threw, through)
Ans: Threw. The ball was thrown by her (verb)

Sentence Rearrangement
Sentence rearrangement is one of those sections that are difficult and time-consuming question asked
in Banking exams and other competitive exams. It consists of a group of jumbled sentences that need to be
re-assembled to make sense. It sounds easy but it can get tricky when more than one sentence seems to fit
perfectly. That is where we have to use vocabulary and context to decide the closest answer. Most banking
exams consist of these type of questions and we are here to break it for you:
Approaching Sentence Rearrangement

1. Check the links: There is always something connecting either in the context or in the flow of the
sentence. Usually, it is a particular theme and you can identify.
2. Related pronouns like who, what, which gives you what goes ahead. You can tell whether this will be
followed by a noun or a verb. General knowledge of grammar comes in handy.
3. Vocabulary: Knowing the meaning of the words definitely leads you to the following words. Even if
you don’t know the meaning, break the word down and look at the root word of it to get some
context. There is always a hint there.

Source: Youtube.com

Practice Questions for You


Rearrange the given sentences to form a meaningfully coherent passage and answer the following
Questions.
(A) That those rates no longer hold for about a quarter of those items, and the highest tax bracket is left
with just 50 products, compared to more than 250 in July, is a tacit admission that the initial rates were
worked out in a hurry with maximising revenue being the prime consideration.
(B) The script was altered for the second time in two months but with far greater impact.
(C) Soon after Prime Minister Narendra Modi promised far-reaching changes to simplify the goods and
services tax regime, especially for small businesses and consumers, the GST Council delivered some relief
measures on Friday.
(D) At its meeting in Guwahati, the Council lowered the tax rates on over 200 products, compared to 27
items in its previous meeting and about 100 tweaks since GST rates were finalised in May.
(E) Over 1,200 products and services had been placed in five tax brackets — 0%, 5%, 12%, 18% and 28%.
1) Which of the following should be the FIRST sentence after the rearrangement?
(A) (E)
(B) (C)
(C) (A)
(D) (B)
(E) (D)
2) Which of the following should be the SECOND sentence after the rearrangement?
(A) (E)
(B) (C)
(C) (A)
(D) (B)
(E) (D)
3) Which of the following should be the THIRD sentence after the rearrangement?
(A) (E)
(B) (C)
(C) (A)
(D) (B)
(E) (D)
4) Which of the following should be the FOURTH sentence after the rearrangement?
(A) (E)
(B) (C)
(C) (A)
(D) (B)
(E) (D)
5) Which of the following should be the FIFTH sentence after the rearrangement?
(A) (E)
(B) (C)
(C) (A)
(D) (B)
(E) (D)
Practice Unseen Passages here
Answer:
1. (D)
(B) The script was altered for the second time in two months but with far greater impact.

2. (C)
(C) Soon after Prime Minister Narendra Modi promised far-reaching changes to simplify the goods and
services tax regime, especially for small businesses and consumers, the GST Council delivered some relief
measures on Friday.

3. (E)
(D) At its meeting in Guwahati, the Council lowered the tax rates on over 200 products, compared to 27
items in its previous meeting and about 100 tweaks since GST rates were finalised in May.

4. (A)
(E) Over 1,200 products and services had been placed in five tax brackets — 0%, 5%, 12%, 18% and 28%.
5. (C)
(A) That those rates no longer hold for about a quarter of those items, and the highest tax bracket is left
with just 50 products, compared to more than 250 in July, is a tacit admission that the initial rates were
worked out in a hurry with maximising revenue being the prime consideration.

Unseen Passages
Unseen Passages in Bank exam occupy a major share in the English Section. Also, the Unseen passages with
questions and answers following them constitute one of the highest weightage in the exam. To get
expertise in it, one needs to attempt as many passages with questions and answers so as to get a clear
understanding built up on how to approach the questions and answer them for different types of English
RC or Unseen Passages.

Tips for Unseen Passages


Some of the key tips to crack English Unseen passages:

 Read English Newspaper Editorials and Articles. It will help you get clarity on English Reading
Comprehension passages with questions and answers that you should give to the questions.
 Go on online and offline mediums to find as many unseen passages you can find to practice. Practice
really is the key here.
 After going through the English Reading Comprehension passage with questions visualize the
answers, figure out how should they be. That visualization will lead you further towards the answer
in the passage. Time is the essence here.
 Make a summary of the English RC Passage with questions and answer them deploying your best
efforts. Check your answers and match them with model answers. Please note that your RC Passage
summary should be one third of the passage size
 Check that all the key points are covered in the summary of RC passage
 Improve your reading speed for English Reading Comprehension passages with questions and
answers also
Ideally, a single unseen passage with 5 to 6 questions should not take more than 15 minutes of your time
to answer. If it does, don’t waste more than an extra minute on this.
Read about Multiple Meaning Words or Homonyms here in detail

Let’s exercise a few questions on unseen passages


The culture of nuclear families is in fashion. Parents are often heard complaining about the difficulties in
bringing up children these days.
Too much of freedom in demand, too much independence; overnight parties; excessive extravagance,
splurging pocket money; no time for studies and family all this is a common cry of such families.
Aren’t parents, themselves, responsible for this pitiful state? The basic need of a growing youth is the
family, love, attention, and bonding along with moral values. One should not forget that ‘charity begins at
home’.
Independence and individuality both need to be respected, in order to maintain the sanctity of family.
Children, today are to be handled with tact in order to bridge the ever-widening generation gap.
Only the reasonable demands need to be fulfilled, as there are too many expenses to be met and top many
social obligations to be taken care of by the parents.
Our forefathers lived happily in joint families. Children loved to live with their cousins, learned to adjust
within means. They had perfect harmony between the generations.
There never existed the concept of old-age homes. There was a deep respect for the family elders and love,
care, and concern for the youngsters. Even the minor family differences were solved amicably.

Unseen Passage with Question and Answers


Questions:

1. Mention any two major common concerns of a nuclear family.


2. Who, according to the passage, is responsible for them?
3. Explain the expression ‘charity begins at home’.
4. Describe the atmosphere in joint families.
5. Which word in the passage means ‘Holiness of life’?
Answers:

1. Too much independence and no time for studies and family.


2. Parents themselves.
3. The parent should not forget that it is in giving that one receives.
4. In joint families, children get a friendly atmosphere and they also learn to adjust within means.
5. Sanctity

Spelling
 Spelling tests are also part of the banking exams like IBPS/SBI PO, SO exams etc. In this section, the
knowledge of the vocabulary of the candidate is checked. A few standard types of questions can be
asked in this section. In some questions, a few words will be given and you will be asked to pick
either the correctly spelt word or the word that is misspelt. Let us see with the help of examples.
 Spelling
 There is not much to discuss in this section. The reason being that there are no rules to spelling. Not
those that we should be concerned with. So here we will just state questions and at the end each
section we will see some words that are most likely to appear in these exams. Let us start.
 In each of the questions mentioned below, various options are given. Only one of the options is right
and three are wrong. Pick the option that is correctly spelt.

 Type I
 Q 1: (A) Emancipation (B)Emancipasion (C) Emencipation (D) Emansipation
 Answer: The correct option is A) Emancipation which means the fact or process of being set free
from legal, social, or political restrictions; liberation.
 Q 2: (A) Hindrance (B) Corespondence (C) Insurence (D) Assurence
 Answer: The correct option is A) Hindrance which means a thing that provides resistance, delay, or
obstruction to something or someone.
 Q 3: (A) Measureable (B) Manageable (C) Marriagable (D) Manoevrable
 Answer: The correct option is B) Manageable which means to be able to be controlled or dealt with
without difficulty.
 Q 4: (A) Populas (B) Pompuous (C) Prelious (D) Presumptuous
 Answer: The correct option is D) Presumptuous which means (of a person or their behaviour)
failing to observe the limits of what is permitted or appropriate.
 Q 5: (A) Enterpreneur (B) Entreprenure (C) Entrepreneur (D) Enterprenure
Answer: The answer is C) Entrepreneur that means a person who sets up a business or businesses,
taking on financial risks in the hope of profit.
 Q 6: (A) Prejudice (B) Prejudise (C) Predjudice (D) Prejudis
 Answer: The answer is A) Prejudice which means a preconceived opinion that is not based on
reason or actual experience.
 Q 7: (A) Independance (B) Independence (C) Independens (D) Indepandance
 Answer: The correct answer is B) Independence which means the fact or state of being independent.
 Q 8: (A) Loanly (B) Lonly (C) Loneli (D) Lonely
 Answer: The answer is D) Lonely which means a state of solitude.

 Type II
 In the following questions, four words are given, only one of which is wrongly spelt. Choose the one
that is not right.
 Q 1: (A) Poignant (B) Relevent (C) Prevalent (D) Malignant
 Answer: The option that is wrongly spelt is B) relevent. Therefore the correct spelling is relevant
which means something or someone closely connected or appropriate to what is being done or
considered.
 Q 2: (A) Vandetta (B) Verisimilitude (C) Vicarious (D) Vociferate
 Answer: The option that is wrongly spelt is A) vandetta. Therefore the correct spelling is Vendetta
which means a blood feud in which the family of a murdered person seeks vengeance on the
murderer or the murderer’s family.
 Q 3: (A) Embankment (B) Deliberately (C) Preceding (D) Proleferous
 Answer: The word that is not correctly spelt here is D) Proleferous. Therefore the correct spelling
is proliferous which means something (of a plant) producing buds or side shoots from a flower or
other terminal part.

 Some More Examples


 Q 4: (A) Malignity (B) Malodorus (C) Malfeasance (D) Malevolent
 Answer: The word that is not correctly spelt is B) Malodorus . Therefore the correct way to spell it is
malodorous which means something which is smelling very unpleasant.
 Q 5: (A) Unaccompanied (B) Unaccusstomed (C) Unadopted (D) Unalloyed
 Answer: The word that is not correctly spelt is B) Unaccusstomed. Therefore the correct way to spell
it is Unaccustomed which means not familiar or usual; out of the ordinary.
 Q 6: (A) Boutique (B) Physique (C) Opaque (D) Obleeque
 Answer: The word that is not correctly spelt here is D) Obleeque. Therefore the correct way to spell
it is oblique which means neither parallel nor at right angles to a specified or implied line; slanting.
 Q 7: (A) Admitting (B) Budgetting (C) Preferring (D) Travelling
 Answer: The word that is not correctly spelt here is B) Budgetting. Therefore the correct way to
spell it is Budgeting which means that allow or provide a particular amount of money on a budget.
 Q 8: (A) Successor (B) Marriage (C) Auctioner (D) Enthusiastic
 Answer: The word that is not correctly spelt here is B) Auctioner. Therefore the correct way to spell
it is Auctioneer which means a person who conducts auctions by accepting bids and declaring goods
sold.
 Q 9: (A) Anniversary (B) Bureaucracy (C) Heresy (D) Secresy
 Answer: The word that is not correctly spelt here is D) Secresy. Therefore the correct way to spell it
is Secrecy which means the action of keeping something secret or the state of being kept secret.
 Practice Questions
 Q 1: Pick out the word that is correctly spelt.
 (A) Refference (B) Relliance (C) Occurence (D) Recurrence
 Ans: D) Recurrence
 Q 2: Select the word that is correctly spelt.
 (A) Flabergast (B) Flabergust (C) Flabbergast (D) Fiabbergust
 Ans: C) Flabbergast
 Q 3: Select the word that is not correctly spelt.
 Pneumonia (B) Diarrheoa (C) Xenophobia (D) Amnesia
 Ans: B) Diarrheoa.

Match the column

Match the column in banking exams are pretty common. The best practice is to go through both the
columns before jumping into the exercises. Connect with context if you don’t know by meaning, eliminate
the ones that cannot be matched, look at them at the end. Chances are all that make sense have been
matched up leaving you the ones that you could not match and that’s a small pot to choose from now.
Examples and exercises in a match the column:

Match the Column A with their meanings in Column B

(i) a quarrel or
1. slaving
an argument

(ii) remove something


from inside another thing 2. chaos
using a sharp tool

(iii) strange, mysterious,


3. rummage
difficult to explain

(iv) finish successfully,


4. scrape out
achieve

(v) search for something


5. stumble over, tumble
by moving things around
into
hurriedly or carelessly

(vi) complete confusion


6. accomplish
and disorder

(vii) fall, or step


7. uncanny
awkwardly while walking

8. (to have or get into) a


(viii) working hard
row

The answer to this Match the column is as under:


1. Slaving = > (viii) working hard.
2. Chaos => (vi) complete confusion and disorder.
3. rummage => (v) search for something by moving things around hurriedly or carelessly.
4. scrape out => (ii) remove something from inside another thing using a sharp tool.
5. stumble over, tumble into => (vii) fall, or step awkwardly while walking.
6. accomplish => (iv) finish successfully, achieve.
7. uncanny => (iii) strange, mysterious, difficult to explain.
8. (to have or get into) a row => (i) a quarrel or an argument.
Sometimes when you don’t know the meaning of the words, try to look at the formation of the word and
you may find a root word that gives you some context of the meaning.
Learn more about Odd One Out here in detail.
For example, if the word is an illusion and you have no clue what it means, try to break the word and you
will see it has ill which can mean something negative like not correct, wrongful, not true and so on.
So, let’s try one example where you can understand this further by attempting and then checking out the
answer later.

Practice Questions

Match The Words with Meanings


All you have to do is to match the words (SET – 1) with their meanings (SET – 2).
SET – 1

1. OBITUARY – ( noun )
2. SWAY – ( verb )
3. MULTINATIONAL – ( adjective )
4. LUXURY – ( noun )
5. AVID – ( adjective )
6. MANAGER – ( noun )
7. FORTUNATE – ( adjective )
8. ENTERPRISING – ( adjective )
9. AMITY – ( noun )
10. ILLUSION – ( noun )
SET – 2

1. Lucky
2. Friendship – a good relationship
3. An open box from which cattle and horse feed
4. A report in a newspaper giving the news of someone’s death
5. Good at thinking of and doing new and difficult things
6. Existing in or involving many countries
7. Something expensive and pleasant but not necessary
8. Very interested or passionate about something
9. An idea or belief which is not true
10. To move slowly from side to side
The Answer to this is:

1. OBITUARY = A report in a newspaper giving the news of someone’s death


2. SWAY = To move slowly from side to side
3. MULTINATIONAL = Existing in or involving many countries
4. LUXURY = Something expensive and pleasant but not necessary
5. AVID = Very interested or passionate about something
6. MANAGER = An open box from which cattle and horse feed
7. FORTUNATE = Lucky
8. ENTERPRISING = Good at thinking of and doing new and difficult things
9. AMITY = Friendship – a good relationship
10. ILLUSION = An idea or belief which is not true

 Theme Detection
 Passage Completion
 Sentence Completion
Sentence completion questions under the English language, are an important part of Competitive exams
for various categories like Bank PO, MBA, CA, RRB, TOFEL, etc. These questions test the candidate’s
vocabulary and knowledge of the finer distinctions among words. There are some strategies that will
greatly help to score on these questions. Commonly these sentences are long and difficult to follow, but
with a little bit practice, one can learn to master them. Let us discuss it in detail.

Sentence Completion
Sentence completions test the skill to use the information observed in complex and incomplete sentences
in order to correctly complete them. It tests a candidate’s vocabulary power and skill to follow the logic of
sentences. These sentences are often quite complex.

Types:
There are possibly four types of sentence completions:
Restatement: Containing words such as namely, in other words, in fact, that is, etc.
Example: The pickpocket was a trickster, in other words, a ______. Here answer will be knave or scoundrel,
which restates “trickster,”
Comparison: Containing the words such as likewise, similarly, and, just as, as like as, etc.
Example: Jack was cleared of all charges; similarly, Jill was ______.
Here we have to compare ‘cleared of all charges’ with the suitable word, and hence vindicated is the
answer.
Contrast: Containing the words such as though, although, however, despite, but, yet, on the other hand, but,
however, despite, or, on the contrary, etc.
Example: Although the tiger is a solitary beast, its cousin the lion is a ______ wild animal.
Here answer should be in contrast with “solitary”. Therefore, gregarious or sociable are possible answers.
Cause and effect: Containing words such as this, therefore, consequently, because of, etc. Also contains
phrases such as due to, as a result, leads to, etc.
Example: A truck stole her parking spot; consequently, Rocky’s ______ look showed her displeasure.
Here answer should be to find the cause for someone to steal. Therefore answer may be scowling or sullen.

Suitable Approaches for Sentence Completion:


A good vocabulary can be a great help for such sentence completion. Although we can use many
approaches for such type of questions, even without knowing all the choices. Some of them are as follows.

1. Read the Sentence:


Use the sentence clues by reading thoroughly. The question may be difficult due to difficult words and the
structure of the sentence. One has to dissect the sentence to figure out what fits best, otherwise one cannot
crack the question though knowing the word meanings.

2. Hints:
The hints given may indicate what should go into the blank for the meaningful sentence. Here’s a test to
locate the right hint. If we change the hint, then the choice in the blank must change. We can check the hint
by putting that word or phrase into the blank itself.

3. Pluses and Minuses:


Once we find the word clues, indicate the kind of word we are now looking for with positive meaning or
negative meaning. Also, to indicate synonyms or antonyms, we can use these symbols.

4. Structure Words:
See and try for words like but, rather, although, however, and, while, but, therefore, etc.
These may reveal the sentence organization and the relationship between hint and blank. They tell about
kinds of words to look for as they change the thought process in the sentence.

5. Visualize:
Before going for the choices, think of the possible words for the blanks. It will save us from wrong choices.
If we know roughly the type of words required, the process of elimination becomes much easier. The word
we see doesn’t have to be fancy and a general idea is fine.

6. Elimination:
Ruling out the wrong choices is now easy. But remember that words have to fit in the given order for the
correct answer. If one word is a perfect choice without making sense, then the answer is incorrect. Don’t
rule out choices if don’t know their meanings. If doubts are there, leave it and return to other choices.

7. Working Backwards:
The two-blank questions can be easier as we have more opportunities to eliminate wrong choices. If we
can eliminate a choice based on one word, we don’t need to know the other word. Often, working I
backtracking way works better.

Solved Example on Sentence Completion


Q.1: Suresh’s skin was ______ to burn if he spent too much time in the sun.
(i) Prone
(ii) Eminent
(iii) Erect
(iv) Daunted
Answer: Prone is the correct answer which is an adjective. Its meaning is – a tendency or inclination to
something.
Q.2: The Security officer ______ the crowd to step back from the fire to avoid any mishappening.
(i) Undulated
(ii) Enjoined
(iii) Stagnated
(iv) Delineated
Answer: Enjoined is the correct answer which is a verb. Its meaning is to issue an order or command with
authority.

Vocabulary Practice Questions


 Vocabulary Practice Questions section has questions from the concepts of synonyms, antonyms,
vocabulary test, double synonyms etc. The section will help you get a much-needed practice run of
this section. Let us start our practice of the Vocabulary Practice Questions.
 Vocabulary Practice Questions

 Synonyms
 Directions: In each of the following questions, out of the given alternatives choose the one which
best expresses the meaning of the given word.
 Q1: E A G E R [Stenographers’ Exam, 1994]
 A) Clever B) Enthusiastic C) curious D) Devoted
 Q2: P L A C I D
 A) Clear B) Calm C) Enjoyable D) Dull
 Q3: C O M M E M O R A T E
 A) Boast B) Remember C) Manipulate D) Harmonise
 Q4: D E I F Y [C B I 1995]
 A) Face B) Worship C) Flatter D) Challenge
 Q5: A M E N I T I E S
 A) Pageantries B) Privileges C) Facilities D) Courtesies
 Q6: D I S P A R I T Y
 A) Injustice B) Unlikeness C) Partiality D) Distortion
 Q7: B A R B A R I A N
 A) Arrogant B) Impolite C) Uncivilized D) Unkind
 Q8: L E T H A L [ Hotel Management, 1992]
 A) Unlawful B) Deadly C) Sluggish D) Smooth
 Q9: S E R E N E
 A) Solemn B) Meak C) Delicate D) Calm
 Q10: G R U E S O M E
 A) Hateful B) Painful C) Tragic D) Frightful
 Q11: S E C U R E
 A) Secret B) Comfortable C) Safe D) Independent
 Q12: T Y R A N N Y [Railways, 1991]
 A) Misrule B) Power C) Madness D) Cruelty
 Q13: O B L I G A T O R Y
 A) Useful B) required C) Stubborn D) Agreeable
 Q14: C O N N O I S S E U R [Railways, 1996]
 A) Ignorant B) Interpreter C) Delinquent D) Lover of art
 Q15: I M P E T U O U S
 A) Violent B) Resourceful C) Pleasing D) Rash

 Find Your Answers Here
 Q1: C), Q2: B), Q3: B), Q4: B), Q5: B), Q6: B), Q7: C), Q8: B), Q9: D), Q10: D), Q11: C), Q12: D), Q13: B),
Q14: B), Q15: D)
 Antonyms
 Directions: Each of the following questions consists of a word or phrase which is italicised bold in
the sentence given. It is followed by certain words or phrases. Select the word or phrase which is
closest to the opposite in meaning of the italicised bold word or phrase.
 Q1: He is a very timid person. [N D A, 1992]
 A) Dashing B) Outgoing C) Bold D) Chivalrous
 Q2: Raju often walks to school.
 A) Rarely B) Never C) Always D) Sometimes
 Q3: One can acquire fame only by being truthful, honest and faithful.
 A) Lose B) Deprive C) Forsake D) Surrender
 Q4: The treaty was ratified by the heads of states. [C D S, 1994]
 A) Set aside B) Unsettled C) Destroyed D) Annulled
 Q5: Always avoid late-night jobs.
 A) inspire B) compel C) pursue D) take
 Q6: Mother Teresa devoted her life to the service of the poor and the destitute.
 A) Greedy B) Noble C) Rich D) Extraordinary
 Q7: Many people try to resist reforms in the society. [I E S, 1994]
 A) repel B) welcome C) accept D) fight
 Q8: He was always prepared to refute the idea.
 A) Agree B) Subscribe C) Recommend D) Endorse
 Find Your Answers Here
 Q1: C), Q2: A), Q3: A), Q4: D), Q5: C), Q6: C), Q7: B), Q8: D).
 Vocabulary Test
 Directions: In each question below, a word is given in capitals. Against this word, there are some
alternatives, one of which is either same in meaning (synonym) or opposite in meaning (antonyms)
of the word printed in BOLD TYPE. Find out which of the given alternatives is either a synonym or
an antonym of the word, oriented in capitals.
 Q1: C O N S W Q U E N C E [S. B. I. P. O. ]
 A) Indifference B) Affect C) Serial D) Outcome E) Divergence
 Q2: E N I G M A T I C
 A) Short-sighted B) Learned C) Puzzling D) Displeased
 Q3: V I R T U O U S
 A) Vulgar B) Insincere C) Vicious D) Miserly
 Q4: I M P E T U O U S
 A) Pleasing B) Rash C) Violent D) Resourceful
 Q5: I N T E R I M
 A) Interval B) Temporary C) Timely D) Internal
 Q6: E X P U R G A T E
 A) Admit B) Renew C) Stain D) Entertain
 Q7: A B A T E [Bank P. O. 1996]
 A) Tighten B) Increase C) Abandon D) Diminish E) Postpone
 Q8: T E M P E R A M E N T A L
 A) Satisfied B) Contented C) Unruffled D) Pleased
 Find Your Answers Here
 Q1: D) [Synonym], Q2: C) [Synonym], Q3: C) [Antonym], Q4: B) [Synonym], Q5: B) [Synonym], Q6: A)
[Antonym], Q7: B) [Antonym], Q8: C) [Antonym].
Word Association Pairs
It is a type of test of any language vocabulary. Word association pairs is a way in which words associates
with each other. And it influences the learning and remembering of words. This word association pair is an
important part of the evaluation of English vocabulary knowledge during various competitive
examinations. The term association refers to the connection or relation between words, ideas, and
concepts, which exists in the human mind and manifests in different ways. Let us discuss it in detail.

Word Association Pairs


In Word Association Pairs, candidates are given a set of 4 words, which contain one pair that most nearly
mean the same or the opposite of each other. The candidate has to choose the option with the correct pair.
Traditionally subjects give a stimulus word ask to produce the first response which comes in the mind.
The use of word associations holds a great deal of promise in the areas of language vocabulary research
and measurement. Many examination bodies use word association pair’s procedures as an alternative way
to test vocabulary.
Word association theory looks for latent relationships that words have with other words, images and
thoughts. Sometimes word association pairs system is like a spider web in which words in the mental
network are related to other words.
In order to take test people to use the Word-Association-Pairs test as a strong technique. Psychiatrists also
widely use this technique in psychology. After analyzing the data, they claimed that there was uniformity
in the organization of associations and people shared stable networks of connections.
The underlying assumption in a word association pair is that the order of the response retrieval from long-
term memory reflects at least a significant part of the structure within and between concepts. In a word
association pairs, the degree of overlap of response hierarchies is a measure of the semantic proximity of
the stimulus words.

Some Characteristics
Word association pairs as a means of assessing proficiency in a foreign language have always something of
an inherent appeal to it. Some points should be kept in mind that
1) It should be relatively quick and easy both to conduct and to score,
2) It should be a nice complement to other methods of assessing learner performance,
3) It should tend to suggest that there may be something of a connection between knowledge and language
proficiency.

The approach of Word Association Pair


It’s easy to expertise in the Word Association Pair questions if vocabulary is strong. Especially knowledge
of synonyms and antonyms is very much essential for it.
Synonyms: These are words with exact or similar meaning as that of the given word.
Antonyms: These are the words that have the opposite meaning as that of the given word.
For example:
Synonyms of the word Massive are as follows:
Gigantic, Huge, Giant, Great.
Antonyms of the word Massive are as follows:
Tiny, Small

Tips to improve the skill

 Whenever come across a new word, look up its synonyms & antonyms online and adopt it.
 Learn synonyms and antonyms of common English words on a regular basis.
 Regularly study ‘Word Power Made Easy’ by Norman Lewis to improve your vocabulary
 Thesaurus is the best resource to learn synonyms and antonyms.

Two Approaches:
First Approach: One should read the given words one by one and check if the other words are their
synonyms/antonyms.
Second Approach: One should check the given options (pairs) one by one, and select the option which has a
synonym/antonym pairing. Use this approach when time is running short.
Solved Example on Word Association Pairs
Q.1: Four words are given below. Select the pair which mean the same or opposite of each other:

1. Menial
2. Minuscule
3. Impressive
4. Massive
Answer: Here option 2 is the right answer. Because Meaning of Minuscule is Extremely Small on the other
hand Meaning of Massive is Large/Heavy.
Q.2 Government must not be partial to its citizen. Find the synonym word of the underlined word.

1. Unequal
2. Favoring
3. Fairless
4. Biased
Answer: Here Biased has a similar meaning as Partial. So the answer is Biased.

You might also like